SlideShare ist ein Scribd-Unternehmen logo
1 von 31
CHAPTER 4
                              FOURIER TRANSFORMS
INTEGRAL TRANSFORM
                                                                              b

       The integral transform of a function f (x ) is defined by              ∫ f ( x).k (s , x)dx where
                                                                              a

       k(s , x) is a known function of s and x and it is called the kernel of the transform.
       When k(s , x) is a sine or cosine function, we get transforms called Fourier sine or
cosine transforms.

FOURIER INTEGRAL THEOREM

       If f (x) is a given function defined in (-l , l) and satisfies Dirichlet’s conditions, then
                           ∞ ∞
                             1
                            π ∫ −∫
                f ( x) =           f (t ) cos λ (t − x) dt dλ
                               0 ∞

At a point of discontinuity the value of the integral on the left of above equation is
1
  { f ( x + 0) − f ( x − 0)}.
2

EXAMPLES
                                 1 for x ≤ 1
                                 
1. Express the function f ( x) =                as a Fourier Integral. Hence evaluate
                                 0 for x > 1
                                 
∞                                       ∞
  sin λ cos λx                            sin λ
∫ λ
0
               dλ and find the value of ∫
                                        0
                                            λ
                                                dλ .


Solution:
       We know that the Fourier Integral formula for f (x) is
                              ∞ ∞
                          1
                 f ( x) =
                          π   ∫ ∫ f (t ) cos λ (t − x) dt dλ
                              0 −∞
                                                                                         ……………….(1)

       Here      f (t ) = 1 for      t ≤ 1          i.e.,   f(t) = 1 in -1 < t < 1
                 f (t ) = 0 for t > 1
                 f (t ) = 0 in − ∞ < t < −1 and 1 < t < ∞
                                             ∞ 1
                                         1
∴ Equation (1) ⇒              f ( x) =
                                         π   ∫ ∫ cos λ (t − x) dt dλ
                                             0 −1


                                             ∞                   1
                                         1  sin λ (t − x) 
                                         π ∫
                                     =                     dλ
                                           0
                                                  λ         −1
                                             ∞
                                         1 sin λ (1 − x) − sin λ (−1 − x)
                                         π∫
                                     =                                    dλ
                                          0
                                                          λ
                                             ∞
                                      1 sin λ (1 − x) + sin λ (1 + x )
                                     = ∫                               dλ
                                      π 0             λ


                                                                                                           1
∞
                                             2 sin λ cos λx
                                             π∫
                                ∴ f ( x) =                  dλ                        .………………(2)
                                              0
                                                     λ

                                                      [Using sin (A+B) + sin (A-B) = 2 sin A cos B]

This is Fourier Integral of the given function. From (2) we get

                        ∞
                          sin λ cos λx                 π
                        ∫
                        0
                                λ
                                       dλ =
                                                       2
                                                         f ( x)                      ……………….(3

                                                      
                                                      1 for x ≤ 1
                                  But        f ( x) =                               ………………..(4)
                                                      0 for x > 1
                                                      
Substituting (4) in (3) we get

                                                π
                            ∞
                              sin λ cos λx         for x ≤ 1
                            ∫ λ            dλ =  2
                            0                   0 for x > 1
                                                
                       ∞
                          sin λ      π
Putting x = 0 we get   ∫0
                            λ
                                dλ =
                                     2

2. Find the Fourier Integral of the function
                                  0     x<0
                                  1
                                  
                         f ( x) =        x=0
                                  2
                                  e − x x > 0
                                  
Verify the representation directly at the point x = 0.
Solution:
       The Fourier integral of f (x) is
                                ∞ ∞
                            1
                 f ( x) =
                            π   ∫ ∫ f (t ) cos λ (t − x) dt dλ
                                0 −∞
                                                                                       ……………….(1)

                        1                                                          
                          ∞   0                            ∞
                       =  ∫ −∫∞
                        π 0
                                 f (t ) cos λ (t − x )dt + ∫ f (t ) cos λ (t − x)dt d λ
                                                           0                        
                        1                                                     
                          ∞   0                         ∞
                       = ∫  ∫ 0. cos λ (t − x)dt + ∫ e −t cos λ (t − x)dt dλ
                        π 0 − ∞                        0                      
                          ∞                                                    ∞
                        1  e −t
                       = ∫ 2     [ − cos( λt − λx ) + λ sin(λt − λx)]  dλ
                                                                       
                        π 0 λ +1                                      0

                                  ∞
                                1 cos λx + λ sin λx
                                π∫
                   f (x) =                          dλ                                     ……….………(2)
                                 0     λ2 + 1




                                                                                                      2
Putting x = 0 in (2), we get
                             ∞
                           1
                  f (0) = ∫ 2
                                 1
                          π 0 λ +1
                                           1
                                     d λ = tan −1 ( λ ) 0
                                           π
                                                        ∞
                                                            [           ]
                           1
                                     [
                        = tan −1 ( ∞ ) − tan −1 (0)
                          π
                                                                  ]
                           1 π  1
                        =  =
                          π 2 2
                                             1
The value of the given function at x = 0 is . Hence verified.
                                             2

FOURIER SINE AND COSINE INTEGRALS

       The integral of the form
                             2∞       ∞
                    f ( x) = ∫ sin λx ∫ f (t ) sin λt dt dλ
                            π0        0

is known as Fourier sine integral.

        The integral of the form
                                ∞      ∞
                              2
                     f ( x) = ∫ cos λx ∫ f (t ) cos λt dt dλ
                             π 0       0



is known as Fourier cosine integral.

PROBLEMS

1. Using Fourier integral formula, prove that

                                               2(b 2 − a 2 ) ∞       u sin xu
                                                             ∫ (u 2 + a 2 )(u 2 + b 2 ) du (a, b > 0)
                         − ax        − bx
                     e          −e           =
                                                    π        0



Solution:
       The presence of sin xu in the integral suggests that the Fourier sine integral formula
has been used.
       Fourier sine integral representation is given by

                                         ∞          ∞
                              2
                      f ( x) = ∫ sin ux ∫ f (t ) sin ut dt du
                              π 0       0


                                               ∞
                                                           ∞                               
                                                 sin ux du  ∫ ( e − at − e −bt ) sin ut dt 
                                             2
                    e − ax − e −bx =           ∫
                                             π 0           0                               




                                                                                                        3
∞                                                                                        ∞
                                       2                           e − at                                − bt
                                                                                                                                      
                                      = ∫ sin ux du                2        { − a sin ut − u cos ut} − 2e 2 { − b sin ut − u cos ut} 
                                       π 0                        a + u
                                                                           2
                                                                                                       b +u                           0
                                              ∞
                                           2              u               u 
                                      =      ∫ sin ux du  a 2 + u 2 − b 2 + u 2 
                                           π 0                                  
                                                              ∞
                                        2(b 2 − a 2 )         u sin ux
                                      =
                                             π        ∫ (u 2 + a 2 )(u 2 + b 2 ) du
                                                      0


2. Using Fourier integral formula, prove that

                                        2
                                              ∞
                                                  (λ   2
                                                           + 2 ) cos xλ
                                              ∫                         dλ
                           −x
                       e        cos x =
                                        π     0            λ2 + 4

Solution:
       The presence of cos xλ in the integral suggests that the Fourier cosine integral
formula for e − x cos x has been used.
       Fourier cosine integral representation is given by

                                       ∞               ∞
                                     2
                                     π∫
                       f ( x) =          cos λx ∫ f (t ) cos λt dt dλ
                                       0        0



                              2∞           ∞ − t                
             ∴e   −x
                       cos x = ∫ cos xλ dλ  ∫ e cos t cos λt dt 
                              π 0          0                    

                                     2
                                       ∞
                                                   1 ∞                                      
                                 =     ∫ cos xλ dλ  ∫ e −t { cos(λ + 1)t + cos(λ − 1)t } dt 
                                     π 0           2 0                                      

                                       ∞
                                                    
                                 =
                                     2
                                     π 0            
                                                          1          −t
                                                                             [
                                       ∫ cos xλ dλ  (λ + 1) 2 + 1 e { − cos(λ + 1)t + (λ + 1) sin(λ + 1)t}            ]   ∞
                                                                                                                           0
                                                    
                                          +
                                                  1
                                             (λ − 1) + 1
                                                     2
                                                                    [
                                                         e −t { − cos(λ − 1)t + (λ − 1) sin(λ − 1)t } 0
                                                                                                      ∞
                                                                                                              ]   )
                                     ∞
                                   1         1           1     
                                  = ∫              +            cos xλ dλ
                                   π 0  (λ + 1) + 1 (λ − 1) + 1
                                                2           2




                                          ∞
                                   2 (λ2 + 2) cos xλ
                                  = ∫                dλ.
                                   π 0   λ2 + 4




COMPLEX FORM OF FOURIER INTEGRALS


                                                                                                                           4
The integral of the form

                                ∞              ∞
                            1
                                ∫    e − iλx   ∫ f (t ) e
                                                             iλt
                f ( x) =                                           dt d λ
                           2π   −∞             −∞


is known as Complex form of Fourier Integral.

FOURIER TRANSFORMS
COMPLEX FOURIER TRANSFORMS
                                        ∞
                                    1
         The function F [ f ( x)] =     ∫∞ f (t ).e dt is called the Complex Fourier transform
                                                   ist

                                    2π −
of f (x ) .

INVERSION FORMULA FOR THE COMPLEX FOURIER TRANSFORM
                               ∞
                           1
                               ∫∞F [ f ( x)].e ds is called the inversion formula for the
                                              −isx
     The function f ( x) =
                           2π −
Complex Fourier transform of F [ f ( x)] and it is denoted by F −1 [ F ( f ( x))].

FOURIER SINE TRANSFORMS
                                                        ∞
                                    2
       The function FS [ f ( x )] =                     ∫ f (t ).sin st dt      is called the Fourier Sine Transform of
                                    π                   0

the function f (x ) .

                                               ∞
                             2
       The function f ( x) =
                             π                 ∫ F [ f ( x)]. sin sx ds is called the inversion formula for the
                                               0
                                                    S


Fourier sine transform and it is denoted by FS
                                                                       −1
                                                                            [ FS ( f ( x))].
FOURIER COSINE TRANSFORMS
                                    ∞
                                  2
      The function FC [ f ( x)] =
                                  π ∫
                                      f (t ). cos st dt is called the Fourier Cosine
                                    0

Transform of f (x) .
                                               ∞
                             2
       The function f ( x) =
                             π                 ∫ F [ f ( x)]. cos sx ds
                                               0
                                                    C                               is called the inversion formula for the

Fourier Cosine Transform and it is denoted by FC
                                                                               −1
                                                                                    [ FC ( f ( x))].
PROBLEMS

1. Find the Fourier Transform of
                  1 − x 2 in x ≤ 1
                  
         f ( x) = 
                  0
                          in x > 1




                                                                                                                          5
∞
                                 sin s − s cos s    s     3π
   Hence prove that         ∫
                            0          s 3
                                                 cos ds =
                                                    2     16
                                                             .


Solution:
       We know that the Fourier transform of f (x) is given by

                                             ∞
                                 1
           F [ f ( x )] =                    ∫ f ( x).e
                                                            isx
                                                                  dx
                                 2π      −∞



                                             −1                                   1                                      ∞
                                1                                            1                                  1
                      =                      ∫ f ( x).e           dx +            ∫ f ( x).e         dx +                ∫ f ( x).e
                                                            isx                                isx                                      isx
                                                                                                                                              dx
                                2π       −∞                                  2π   −1                            2π       1
                                         −1                                  1                                      ∞
                                1                                      1                                    1
                     =                   ∫ 0.e dx +                          ∫ (1 − x ).e dx +                      ∫ 0.e
                                              isx                                    2   isx                                 isx
                                                                                                                                   dx
                                2π       −∞                            2π    −1                             2π       1


                                         1
                                1
                     =                   ∫ (1 − x
                                                       2
                                                           ).e isx dx
                                2π       −1



                                                                                                       1
                                1              e isx            e isx   e isx 
                     =              (1 − x 2 )       − ( −2 x ) 2 2 − 2 3 3 
                                2π              is             i s     i s  −1


                                1  − 2 is  2 is − 2 −is 2 e −is 
                     =              2 e + 3e + 2 e −            
                                2π  s     is    s       i s3 


                                1  − 2 is             2                
                     =             s   (e + e −is ) + 3 (e is − e −is )
                                2π 
                                      2
                                                      is                

                                1 − 4           4                                    1 4                       
                     =              s 2 cos s + s 3 sin s  =                             s 3 (sin s − s cos s )
                                2π                                                   2π                        

By using inverse Fourier Transform we get

                                                       ∞
                                1                 1         4
               f ( x) =
                                2π
                                         .
                                                  2π
                                                       ∫
                                                       −∞   s 3
                                                                (sin s − s cos s ).e −isx ds

                                    ∞
                           1             4
                     =
                          2π        ∫s
                                    −∞
                                             3
                                                  (sin s − s cos s ).(cos sx − i sin sx ) ds

                                    ∞
                           1             4
                     =
                          2π        ∫
                                    −∞   s3
                                            (sin s − s cos s ) cos sx ds

                                                                   ∞
                                                          1                4
                                                       −
                                                         2π        ∫
                                                                   −∞      s3
                                                                              (sin s − s cos s ) i sin sx ds



                                                                                                                                                   6
The second integral is odd and hence its values is zero.

                                  ∞
                              2 sin s − s cos s
                              π −∫
          ∴        f ( x) =                     cos sx ds
                                 ∞    s3
                                  ∞
                             4 sin s − s cos s
                            = ∫                cos sx ds
                             π 0     s3

          ∞
            sin s − s cos s            π
 i.e.,    ∫
          0        s 3
                            cos sx ds = f ( x)
                                       4
             1
Putting x = , we get
             2
         ∞
             sin s − s cos s     s     π  1  π  1  3π
         ∫0         s  3
                             cos ds =
                                 2
                                         f   = 1 −  =
                                        4  2  4  4  16
                                                           .

          ∞
              sin s − s cos s    s     3π
          ∫
          0         s 3
                              cos ds =
                                 2     16
                                          .


2. Find the Fourier sine transform of e − x , x ≥ 0 (or) e − x , x > 0. Hence evaluate
    ∞
      x sin mx
    ∫ 1 + x 2 dx.
    0

Solution:
       The Fourier sine transform of f(x) is given by

                                          ∞
                              2
              FS [ f ( x )] =             ∫ f ( x).sin sx dx
                              π           0
         −x        −x
Here e        =e        for x > 0

                                      ∞
                   [ ]
              FS e − x =
                                  2
                                  π   ∫e
                                              −x
                                                   . sin sx dx
                                      0

                              2 s                                ∞ − ax             b 
                            =                                     ∫ e sin bx dx = 2     
                              π s2 +1                            0               a + b2 
Using inverse Fourier sine transform we get

                                  ∞

                                  ∫ F [e ]. sin sx ds
                              2               −x
              f ( x) =                s
                              π   0


                                  ∞
                              2           2     s
                        =
                              π   ∫
                                  0
                                            . 2
                                          π s +1
                                                  . sin sx ds

                              ∞
                            2      s
                        =     ∫ s 2 + 1 sin sx ds
                            π 0


                                                                                             7
∞
                 π              s
      i.e.,        f ( x) = ∫ 2    . sin sx ds
                 2          0 s +1
                ∞
                      s. sin sx     π
      i.e.,     ∫0     s +1
                         2
                                ds = e − x
                                    2

Replacing x by m we get

                         ∞
                             s. sin ms     π
              i.e.,      ∫
                         0    s +1
                                 2
                                       ds = e − m
                                           2

                         ∞
                             x. sin mx     π
              i.e.,      ∫
                         0    x +1
                                 2
                                       dx = e −m
                                           2
                                                                      [since s is dummy variable, we can replace it by x]


                                                                      e − ax
3. Find the Fourier cosine transform of                                      .
                                                                        x
Solution:
                                                                 ∞
                                                             2
              We know that FC [ f ( x )] =                       ∫ f ( x). cos sx dx
                                                             π   0
                                            − ax
                                        e
Here                         f ( x) =              .
                            x
                                                   ∞
                            2                        e − ax
          ∴ FC [ f ( x )] =                        ∫ x . cos sx dx
                            π                      0



Let                   FC [ f ( x)] = F ( s )

                                                   ∞
                                            2        e − ax
Then                         F (s) =
                                            π      ∫ x . cos sx dx
                                                   0
                                                                                                                   ………………(1)


Differentiating on both sides w.r.t. ‘s’ we get,

                                                       ∞
                       dF ( s ) d 2                      e − ax
                        ds
                               =
                                 ds π                  ∫ x . cos sx dx
                                                       0
                                         ∞
                                        2 ∂  e −ax       
                                =        ∫ ∂s  x . cos sxdx
                                        π0               
                                             ∞                                       ∞
                                        2 e −ax                  2 − ax
                                =        ∫ x (− sin sx).x dx = − π ∫ e sin sx dx
                                        π0                         0

                       dF ( s )    2     s                                      ∞
                                                                                                                b 
                                                                                 ∫e
                                                                                         − ax
                                =−   . 2                                                      sin bx dx =          
                        ds         π a + s2                                     0                            a + b2 
                                                                                                               2




Integrating w.r.t. ‘s’ we get


                                                                                                                               8
2      s
            F (s) = −      .∫ 2   ds
                         π s + a2

                        2 1                        1
                 =−      . . log ( s 2 + a 2 ) = −    . log ( s 2 + a 2 )
                        π 2                        2π

                                                                 e − ax − e −bx
4. Find the Fourier cosine transform of                                         .
                                                                        x
Solution:
       We know that the Fourier cosine transform of f(x) is

                                            ∞
                                      2
               FC [ f ( x )] =              ∫ f ( x). cos sx dx
                                      π         0
                                     − ax
                                 e          − e −bx
Here                 f ( x) =
                                            x

                                            ∞
         e − ax − e −bx   2                 e − ax − e − bx
   ∴ FC 
                x
                         =
                           π               ∫ x . cos sx dx
                                            0


                                            ∞                                  ∞
                             2                e − ax          2 e −bx
                           =
                             π              ∫ x . cos sx dx − π ∫ x cos sx dx
                                            0                   0




                                 e −ax        e −bx 
                           = Fc         − Fc        
                                 x            x 

                                            1                             1
                           =−                       log ( s 2 + a 2 ) +            log ( s 2 + b 2 )
                                            2π                            2π


                                      1       s2 + b2 
                           =             log  2
                                              s + a2 
                                      2π              

                                                            e − as
5. Find f (x) , if its sine transform is                           . Hence deduce that the inverse sine
                                                              s
               1
transform of     .
               s
Solution:
       We know that the inverse Fourier sine transform of FS [ f (x )] is given by
                                                ∞
                              2
                     f ( x) =
                              π                 ∫ F [ f ( x)]. sin sx ds
                                                0
                                                    S


                                 e − as
Here           FS [ f ( x )] =
                                   s


                                                                                                          9
∞
                                      2     e − as
            ∴          f ( x) =
                                      π   ∫ s . sin sx ds
                                          0



Differentiating w.r.t. ‘x’ on both sides, we get,

                 d [ f ( x )]
                                              ∞
                                2               e − as ∂
                     dx
                              =
                                π             ∫ s . ∂x (sin sx) ds
                                              0
                                               ∞                                    ∞
                                          2      e − as            2 − as
                                  =
                                          π    ∫ s . cos sx s ds = π ∫ e . cos sx ds
                                               0                     0

                                          2    a                   ∞ −ax              a 
                                  =                                ∫ e cos bx dx = 2     
                                          π a + x2
                                             2
                                                                   0               a + b2 
                  d [ f ( x )]   2    a
                               =
                      dx         π x + a2
                                    2


                                              2       1            2 1    −1  x 
                  ∴ f ( x) = a
                                              π ∫ x 2 + a 2 dx = a π a tan  a 
                                                                              
                                          2         x
                  ∴ f ( x) =                tan −1  
                                          π        a
                                                                   1
To find the inverse Fourier sine transform of                        :
                                                                   s
           Put a = 0, in (1), we get

                         2              2 π  π
            f ( x) =       tan −1 (∞) =  . =
                         π              π 2  2

PROPERTIES

 1. Linearity Property
       If F(s) and G(s) are the Fourier transform of f (x ) and g (x) respectively then
                 F [ a f ( x ) + b g ( x)] = a F ( s) + b G ( s )
Proof:
                                            ∞
                                       1
        F [ a f ( x) + b g ( x)] =          ∫∞[ a f ( x) + b g ( x)] e dx
                                                                      isx

                                       2π −

                                                   ∞                                     ∞
                                              1                                 1
                                      =            ∫ a f ( x).e dx +                     ∫ b g ( x).e
                                                               isx                                      isx
                                                                                                              dx
                                              2π   −∞                           2π      −∞



                                                   ∞                                ∞
                                              a                             b
                                      =            ∫    f ( x).e isx dx +           ∫ g ( x).e
                                                                                                 isx
                                                                                                       dx
                                              2π   −∞                       2π      −∞



                                      = a F ( s ) + b G ( s)



                                                                                                                   10
2. Change of Scale Property

                                                                                                                         1 s
         If F(s) is the Fourier transform of f (x ) then F [ f (ax)] =                                                    F  ,a>0
                                                                                                                         a a
Proof:
                                           ∞
                                  1
         F [ f (ax)] =                     ∫ f (ax).e
                                                              isx
                                                                     dx
                            2π             −∞
Put                   ax = y
                                                                       dy
                     a dx = dy                 i.e., dx =
                                                                       a
When x = −∞,            y = −∞ and x = ∞,                                   y=∞

                                           ∞                     y                             ∞             s
                                  1                                    dy 1            1                    i  y
         F [ f (ax)] =
                                                            is

                                  2π
                                           ∫ f ( y).e
                                           −∞
                                                                 a
                                                                     .   =
                                                                       a a             2π
                                                                                               ∫ f ( y).e
                                                                                              −∞
                                                                                                             a
                                                                                                                     .dy


                              1
                         =      F ( s a)
                              a

3. Shifting Property ( Shifting in x )

         If F(s) is the Fourier transform of f (x ) then F [ f ( x − a )] = e ias F ( s )
Proof:
                                                 ∞
                                       1
         F [ f ( x − a)] =                       ∫ f ( x − a).e
                                                                            isx
                                                                                  dx
                                       2π       −∞


Put     x-a = y
         dx = dy
When x = −∞, y = −∞ and x = ∞,                                              y=∞

                                                 ∞                                                 ∞
                                       1                                                   e ias
         F [ f ( x − a)] =                       ∫   f ( y ).e is ( y + a ) . dy =                 ∫ f ( y).e
                                                                                                                isy
                                                                                                                         .dy
                                       2π       −∞                                          2π     −∞



                                                ∞
                                  e ias
                              =                 ∫ f ( x).e             .dx = e isa F ( s )
                                                                 isx

                                      2π        −∞


4. Shifting in respect of s

         If F(s) is the Fourier transform of f (x ) then F e iax f ( x) = F ( s + a )               [                ]
Proof:
                                                ∞
         Fe[   iax
                          ]
                     f ( x) =
                                      1
                                                ∫e
                                                     iax
                                                           f ( x) e isx dx
                                      2π       −∞




                                                                                                                                      11
∞
                              1
                                        ∫ f ( x).e
                                                         i( s+a) x
                        =                                            dx = F ( s + a)
                              2π     −∞


5. Modulation Theorem
                                                                                                             1
         If F(s) is the Fourier transform of f (x ) then F [ f ( x) cos ax ] =                                 [ F ( s + a ) + F ( s − a )]
                                                                                                             2
Proof:
                                            ∞
                                    1
         F [ f ( x) cos ax ] =              ∫ f ( x). cos ax.e
                                                                         isx
                                                                               dx
                                    2π      −∞



                                            ∞
                                    1                   e iax + e −iax             
                            =           ∫
                                    2π −∞
                                          f ( x).e isx 
                                                       
                                                              2
                                                                                    dx
                                                                                    
                                                                                    

                                                     ∞                                     ∞
                               1 1                                      1 1
                                                  ∫∞ f ( x).e dx + 2 . 2π                  ∫ f ( x).e
                                                             i( s+a ) x                                 i ( s −a ) x
                              = .                                                                                      dx
                               2 2π              −                                         −∞



                                  1              1             1
                              =     f ( s + a ) + f ( s − a ) = [ f ( s + a ) + f ( s − a )]
                                  2              2             2


                                  1
         F [ f ( x) cos ax ] =      [ F ( s + a ) + F ( s − a)]
                                  2

COROLLARIES
                             1
(i ) FC [ f ( x) cos ax ] =    [ FC ( s + a) + FC ( s − a)]
                             2
                             1
(ii ) FC [ f ( x) sin ax ] = [ FS (a + s) + FS (a − s )]
                             2
                               1
(iii ) FS [ f ( x) cos ax ] = [ FS ( s + a ) + FS ( s − a )]
                               2
                             1
(iv ) FS [ f ( x) sin ax ] = [ FC ( s − a ) − FC ( s + a)]
                              2

6. Conjugate Symmetry Property

         If F(s) is the Fourier transform of f (x ) then F f ( − x) = F ( s)           [        ]
Proof:
                                                         ∞
                                                 1
         We know that F ( s ) =                          ∫ f ( x). e
                                                                         isx
                                           dx
                                 2π −∞
Taking complex conjugate on both sides we get
                     ∞
                 1
                     ∫∞ f ( x). e dx
                                 −isx
         F (s) =
                 2π −


                                                                                                                                         12
Put     x = -y
      dx = -dy
When x = −∞, y = ∞ and x = ∞,                                  y = −∞

                                                −∞
                                       1
                ∴ F (s) =                        ∫ f (− y) .e              (− dy )
                                                                     isy

                                       2π        ∞



                                                      −∞
                                           1
                               =−                     ∫ f (− y). e
                                                                           isy
                                                                                 dy
                                           2π         ∞




                                                                                          [            ]
                                                 ∞
                                       1
                               =                 ∫ f (− x). e              dx = F f (− x)
                                                                     isx

                                       2π       −∞


7. Transform of Derivatives

         If F(s) is the Fourier transform of f (x ) and if f (x) is continuous, f ′(x) is piecewise
continuously differentiable, f (x ) and f ′(x) are absolutely integrable in (−∞ , ∞) and
 lim [ f ( x)] = 0 , then
x → ±∞

                F ( f ′( x ) ) = −is F ( s )
Proof:
         By the first three conditions given, F { f (x)} and F { f ′(x)} exist.
                                            ∞
                                   1
              F { f ′( x)} =                ∫ f ′( x) e
                                                               isx
                                                                     dx
                                   2π      −∞



                                                                                      ∞
                           =
                                   1
                                           [e   isx
                                                           ]   ∞
                                                      f ( x) −∞ −
                                                                             is
                                                                                      ∫e
                                                                                              isx
                                                                                                    f ( x) dx, on int egrating by parts.
                                   2π                                        2π       −∞



                           = 0 − isF { f ( x)} , by the given condition.


                           = −is F ( s).

The theorem can be extended as follows.

       If f , f ′, f ′′, , f ( n −1) are continuous, f (n ) is piecewise continuous, f , f ′, f ′′, , f ( n )
are absolutely integrable in (−∞ , ∞) and f , f ′, f ′′,  , f ( n −1) → 0 as x → ±∞ , then

                       F ( f ( n ) ( x) ) = (−is ) n F ( s )




                                                                                                                                           13
8. Derivatives of the Transform

                                                                                          dF ( s )
         If F(s) is the Fourier transform of f (x ) then F [ x. f ( x )] = (−i )
                                                                                           ds
Proof:
                                            ∞
                                   1
                       F (s) =              ∫ f ( x )e
                                                         isx
                                                               dx
                                   2π       −∞



                                            ∞

                                            ∫ ds [ f ( x)e ]dx
                     dF ( s )      1          d
         ∴                    =                                 isx

                      ds           2π       −∞



                                            ∞
                                   i
                               =            ∫ [ x. f ( x)]e          dx = iF [ xf ( x)]
                                                               isx

                                   2π       −∞



                     dF ( s)
         ∴ ( −i )            = F [ x. f ( x)]
                      ds


                           [            ]
Extending, we get, F x n . f ( x) = (−i ) n
                                                      d n F (s)
                                                        ds n

DEFINITION

                 ∞
             1
               ∫ f ( x − u ) g (u )du is called the convolution product or simply the convolution
           2π −∞
of the functions f (x) and g (x) and is denoted by f ( x) * g ( x ) .

9. Convolution Theorem

        If F(s) and G(s) are the Fourier transform of f (x ) and g (x) respectively then the
Fourier transform of the convolution of f(x) and g(x) is the product of their Fourier
transforms.
        i.e., F [ f ( x ) * g ( x)] = F ( s ).G ( s)
Proof
                                      ∞
                                  1
         F [ f ( x ) * g ( x )] =     ∫∞ f ( x) * g ( x)e dx
                                                         isx

                                  2π −
                                   1       1 ∞
                                            ∞
                                                                         isx
                               =       ∫∞  2π −∫∞ f ( x − u) g (u )du  e dx
                                   2π −                                
                                   1
                                       ∞
                                                 1 ∞                     
                               =       ∫∞  2π −∫∞ f ( x − u)e dxdu,
                                                                    isx
                                         g (u ) 
                                   2π −                                   
                                      on changing the order of int egration.




                                                                                                     14
∞

                                                    ∫ g (u )[e                         ]
                                           1
                               =                                      ius
                                                                            F ( s) du , by the shifting property.
                                         2π         −∞
                                                                  ∞
                                                         1
                               = F ( s).                          ∫ g (u ).e
                                                                                     ius
                                                                                           du
                                          2π                    −∞

                               = F ( s).G ( s )
Inverting, we get

          F −1 [ F ( s ).G ( s)] = f ( x) * g ( x )
                                     = F −1 { F ( s )} * F −1 { G ( s )}

10. Parseval’s Identity (or) Energy Theorem

        If f (x) is a given function defined in (−∞ , ∞) then it satisfy the identity,
                               ∞                               ∞

                               ∫                                ∫ F ( s)
                                               2                                 2
                                    f ( x) dx =                                      ds
                            −∞                                −∞

where F(s) is the Fourier transform of f (x ) .
Proof:
       We know that F −1 [ F ( s ).G ( s)] = f ( x) * g ( x )

                           ∞                                                               ∞
                     1                                                           1
                           ∫ F (s).G(s)e                                                   ∫ f (t ) g ( x − t )dt
                                                         −isx
                                                                ds =
                2π −∞                                                            2π        −∞
Putting x = 0, we get

                                    ∞                                       ∞

                                    ∫ F (s).G(s)ds = ∫ f (t ) g (−t )dt
                                    −∞                                      −∞
                                                                                                                            ………………..(1)

                               Let                       g ( − t ) = f (t )                                                 .……………….(2)
                                   i.e.,                     g (t ) = f (−t )                                               ………………..(3)
                                                   ∴ G ( s ) = F [ f (− x)] = F ( s )                               by property (9)
                               i.e.,           ∴ G ( s) = F ( s)                                                            ………………..(4)

        Substituting (2) and (4) in (1) we get

                                    ∞                                        ∞

                                    ∫ F (s).F (s) ds =
                                    −∞
                                                                             ∫ f (t ). f (t ) dt
                                                                            −∞




                                                                                                             [ F (s).F (s) = F (s) ]
                                               ∞                             ∞

                                               ∫                             ∫
                                                              2                                 2                                     2
                                                   F ( s) ds =                       f ( x) dx
                                           −∞                               −∞



11. If f (x) and g (x) are given functions of x and FC [ f ( x)] and FC [ g ( x)] are their
Fourier cosine transforms and FS [ f ( x)] and FS [ g ( x)] are their Fourier sine transforms then




                                                                                                                                          15
∞                          ∞                                    ∞

(i)     ∫
        0
            f ( x ) g ( x)dx = ∫ FC [ f ( x)].FC [ g ( x)]ds = ∫ FS [ f ( x )].FS [ g ( x)]ds
                                   0                                    0
        ∞                     ∞                           ∞

        ∫   f ( x) dx = ∫ FC [ f ( x)] ds = ∫ FS [ f ( x )] ds ,
                    2                            2                          2
(ii)
        0                      0                           0

which is Parseval’s identity for Fourier cosine and sine transforms.

Proof:
                ∞                                         ∞
                                                                  2∞                   
(i)             ∫ FC [ f ( x )].FC [ g ( x )]ds = ∫ FC [ f ( x)]    ∫ g ( x) cos sx dx  ds
                0                                 0               π 0                  

                                                           ∞
                                                                     2∞                          
                                                         = ∫ g ( x)     ∫ FC [ f ( x)] cos sx ds  dx,
                                                           0         π 0                         
                                                                      Changing the order of integration
                                                           ∞
                                                         = ∫ f ( x) g ( x)dx
                                                           0

Similarly we can prove the other part of the result.
(ii) Replacing g ( x) = f * ( x) in (i) and noting that FC [ f ( x)] = FC [ f ( x)] and
FS [ f ( x)] = FS [ f ( x)] , we get
                 ∞                          ∞                                   ∞

                 ∫0
                        f ( x ). f ( x).dx = ∫ FC [ f ( x)].FC [ f ( x)] ds = ∫ FS [ f ( x )].FS [ f ( x )] ds
                                             0                                  0


                    ∞                   ∞                           ∞

                    ∫    f ( x) .dx = ∫ FC [ f ( x)] ds = ∫ FS [ f ( x)] ds
                               2                               2                2
i.e.,
                    0                   0                           0



12. If FC [ f ( x)] = FC ( s ) and FS [ f ( x)] = FS ( s ) , then
     d
(i)     { FC ( s)} = − FS { xf ( x)} and
     ds
     d
(ii)    { FS ( s)} = − FC { xf ( x)}.
     ds
Proof:
                                        ∞
                                      2
                                      π∫
                        FC ( s) =         f ( x) cos sx dx
                                        0



                                                 ∞
                            d
                               { FC ( s)} = ∫ f ( x)(− x sin sx)dx
                            ds              0
                                                     ∞
                                            = − ∫ {xf ( x)}sin sx dx
                                                     0



                              = − FS {xf ( x)}
Similarly the result (ii) follows.


                                                                                                                 16
PROBLEMS

                                                   a 2 − x 2
                                                                   x <a
1. Show that the Fourier transform of f ( x ) =                            is
                                                   0
                                                                   x >a>0
                                                ∞
   2  sin as − as cos as                        sin t − t cos t     π
2                         . Hence deduce that ∫                 dt = . Using Parseval’s
   π          s 3
                                               0       t 3
                                                                      4
                          ∞                           2
                      sin t − t cos t      π
identity show that ∫          3        dt = .
                   0                  
                             t               15

Solution:
       We know that
                                   ∞
                              1
         F [ f ( x )] =            ∫ f ( x).e
                                                isx
                                                      dx
                              2π   −∞




                              1                                               
                                   −a              a             ∞
                    =             ∫ f ( x)e dx + ∫ f ( x)e dx + ∫ f ( x )e dx 
                                            isx            isx             isx

                              2π  −∞             −a             a             


                              1                                  
                                       a                                             a
                                                                            1
                    =             0 + ∫ ( a 2 − x 2 )e isx dx + 0 =                ∫ (a
                                                                                            2
                                                                                                − x 2 ).e isx dx
                              2π  − a                                     2π       −a



                                                                                                    a
                              1  2            e isx                e isx   e isx 
                    =             ( a − x 2 )
                                               is          − (−2 x) 2 2  − 2 3 3 
                                                                    i s  i s 
                              2π                                                  − a


                    =
                              1  − 2a isa
                                  2 e +e [−isa  2i
                                                            ]      [
                                                                   
                                                + 3 e −isa − e isa              ]
                              2π  s             s                 

                              1  − 2a             4        
                     =            2 [2 cos as ] + 3 sin as 
                              2π  s              s         

                              4  sin as − as cos as 
                     =
                              2π 
                                         s3         
                                                     

                      2  sin as − as cos as 
             F (s) = 2
                      π         s3         
                                             
                                 2  sin s − s cos s 
       When a = 1, F ( s ) = 2                                                                              ………………..(A)
                                 π       s3        
                                                     
Using inverse Fourier Transform, we get




                                                                                                                      17
∞
                            1          2            1
             f ( x) =
                            2π
                                 .2.
                                       π       ∫s
                                               −∞
                                                    3
                                                        [sin as − as cos as].e −isx ds


                                           ∞
                            1          2 1
                                                { sin as − as cos as}{ cos sx − i sin sx} ds
                                       π −∫ s 3
                    =            .2.
                            2π            ∞



                             ∞
                        2 sin as − as cos as
                        π −∫
             f ( x) =                        cos sx ds
                           ∞      s3

[The second integral is odd and hence its value is zero]

                            ∞
                    4 sin as − as cos as
                  = ∫                    cos sx ds
                    π 0        s3
[since the integrand is an even function of s]

Putting a = 1, we get
                      ∞
                    4 sin s − s cos s
            f ( x) = ∫                cos sx ds
                    π 0     s3

                            ∞
                     4 sin t − t cos t
            f ( x) = ∫                 cos tx dt
                    π 0      t3
Putting x = 0, in the given function we get

                ∞
            4 sin t − t cos t
            π∫
                              dt = f (0) = 1
             0      t3
                ∞
                  sin t − t cos t      π
            ∴   ∫
                0       t 3
                                  dt =
                                       4
                                           ∞                     ∞

                                           ∫                     ∫ F (s)
                                                            2              2
Using Parseval’s identity,                      f ( x) dx =                    ds   [Using (A)]
                                         −∞                      −∞
                                                        2
                   2                   
             ∞  2.   (sin s − s cos s)       1
                    π
             ∫ 
            − ∞          s3
                                         ds = ∫ (1 − x 2 ) 2 dx
                                              −1
                                       
                                       
                        ∞                               2        1
                    8  sin s − s cos s 
                     ∫∞                 ds = 2 ∫ (1 − x ) dx
                                                         2 2

                    π−       s 3
                                                0
                        ∞                               2
                    16  sin s − s cos s           8
                      ∫
                    π 0       s 3        ds = 2.
                                                  15




                                                                                                  18
∞                           2
                              sin s − s cos s       π
                i.e.,       ∫
                            0       s 3        ds =
                                                     15
                             ∞                       2
                               sin t − t cos t       π
                i.e.,        ∫ t3
                             0
                                                 dt =
                                                      15

2. Find the Fourier Transform of f (x) if
                  1 − x , x < 1
                  
         f ( x) = 
                  0,
                          x >1
                                ∞              4
                           sin t          π
Hence deduce that ∫               dt =
                         0
                              t            3
Solution:
We know that
                                 ∞
                            1
          F [ f ( x )] =         ∫∞ f ( x).e dx
                                            isx

                            2π −

                                         1

                                         ∫ (1 − x ).e
                                    1
                            =                           isx
                                                              dx
                                    2π   −1



Since x > 1, f ( x) = 0, i.e., in − ∞ < x < −1, and 1 < x < ∞, f ( x ) = 0.
                                          1

                                          ∫ [1 − x ] [ cos sx + i sin sx] dx
                                    1
                             =
                                    2π    −1



                                          1

                          ∫ [1 − x ] cos sx dx
                                    1
                             =
                       2π −1
The second integral becomes zero since it is an odd function.
                                          1
                                    2
                             =
                                    2π
                                          ∫ (1 − x) cos sx dx
                                          0

                                             [ [1 − x ] cos x is an even function]
                                                                               1
                               2                  sin sx    − cos sx 
                             =           (1 − x ) s  − (−1) s 2 
                               π                                      0


                                    2  − cos s 1 
                             =                + 2
                                    π  s2      s 


                                    2 1
        i.e.,           F ( s) =     . (1 − cos s )
                                    π s2


                                                                                      19
Using Parseval’s identity

            ∞                             ∞

            ∫                             ∫ F ( s)
                             2                         2
                 f ( x) dx =                               ds
            −∞                            −∞



            1                                  ∞
               [1 − x ] dx = 2 ∫ (1 − cos s) ds
                                            2

            ∫
                             2

            −1
                             π −∞     s4
                 1                                 ∞
                                           2 (1 − cos s) 2
            2 ∫ [1 − x ] dx =
                                           π −∫
                                 2
                                                           ds
                 0                            ∞   s4

                                               ∞
                                     2 2 (1 − cos s ) 2
                                     3 π −∫
                                      =                 ds
                                          ∞   s4


             put              s = 2t                    when s = ∞, t = ∞
                             ds = 2dt                   when s = −∞, t = −∞

                                     π ∞ (1 − cos 2t ) 2
                                     3 −∫
                                      =                  .2 dt
                                        ∞    16t 4


                                      π ∞ (1 − cos 2t ) 2
                                      3 −∫
                                       =                  dt
                                         ∞     8t 4

                                           ∞
                                      π      (1 − cos 2t ) 2
                                        = 2∫                 dt
                                      3    0      8t 4


                                      π ∞ sin 4 t
                                      3 ∫ t4
                                       =          dt
                                         0



                     ∞                4
                        sin t   π
    i.e.,            ∫  t  dt = 3
                     0        

                         ∞
                                         dx
3. Evaluate              ∫ (x
                         0
                                 2
                                     + a )( x 2 + b 2 )
                                          2             using transforms.


Solution:
                                                                                       2     a
      We know that the Fourier cosine transform of f ( x) = e − ax is                    . 2    .
                                                                                       π s + a2
                                                                              2     b
Similarly the Fourier cosine transform of f ( x) = e − ax is                    . 2    .
                                                                              π s + b2


                                                                                                    20
∞                                       ∞

We know that           ∫ FC [ f ( x)].FC [ g ( x)] ds = ∫ f ( x).g ( x) dx
                       0                                       0
                   ∞                                                ∞
                           2     a     2   b
    i.e.,          ∫
                   0
                             . 2     . . 2
                           π s +a π s +b
                                   2         2
                                               . ds = ∫ e − ax .e −bx dx
                                                      0



                                   ∞                                 ∞
                               2             ab
                                 ∫ (s 2 + a 2 )(s 2 + b 2 ) ds = ∫ e
                                                                     −( a +b ) x
    i.e.,                                                                        ds
                               π 0                               0



                                                                    ∞
                                                      e −( a + b ) x            1       1
                                                    =                 = 0−           =
                                                      − ( a + b)  0        − ( a + b) a + b

                   ∞
                                   dx             π
    i.e.,          ∫ (x
                   0
                           2      2    2   2
                                             =
                               + a )( x + b ) 2ab( a + b)

4. Find the Fourier transform of e − a x and hence deduce that
    ∞
       cos xt    π −a x
(i) ∫ 2 2 dt = e
    0 a +t
                2a

      [
(ii) F xe
            −a x
                   ]=i         2     2as
                               π (s + a 2 ) 2
                                   2


Solution:
                                       ∞
                                  1
          F [ f ( x )] =               ∫ f ( x).e
                                                    isx
                                                          dx
                                  2π   −∞




                                 1                                        
                                       0                  ∞
                           =        ∫   f ( x)e isx dx + ∫ f ( x)e isx dx 
                                 2π  −∞                  0                


                                         e − ax
                                                         if 0 ≤ x < ∞
          Here                 f ( x ) =  ax
                                         e
                                                         if − ∞ < x < 0


                                 1  ax isx                          
                                       0           ∞
                           =        ∫   e. e dx + ∫ e −ax .e isx dx 
                                 2π  −∞           0                 


                                 1  ( a +is ) x                          
                                       0              ∞
                           =         ∫ e.       dx + ∫ e −( a −is ) x dx 
                                 2π  −∞              0                   




                                                                                                21
1  e ( a +is ) x     e −( a −is ) x  
                                            0                   ∞

                  =                      +                  
                       2π  (a + is)  −∞  − (a − is )  0 
                                                                 

                       1  1           1 
                  =        a + is + a − is 
                       2π                  


        Fe[   −a x
                      ]=    2    a
                            π s + a2
                               2



Using inversion formula, we get

                                    ∞
                                1        2     a
              f ( x) =
                            2π
                                    ∫
                                    −∞
                                           . 2
                                         π s +a  2
                                                   e −isx ds


                                ∞
                           a cos sx − i sin sx
                           π −∫
                       =                       ds
                              ∞  s2 + a2

                                ∞
                           a      cos x
                       =
                           π−∫∞ s 2 + a 2 ds

              ∞
                  cos sx       π           π −a x
              ∫s
              0
                   2
                     +a 2
                          dx =
                               2a
                                  f ( x) =
                                           2a
                                              .e                       (or )


              ∞
                  cos tx       π −a x
              ∫s
              0
                  2
                    +a  2
                          dt =
                               2a
                                  .e


Putting a = 1, we get,


                     Fe[ ]=−x           2 1
                                         .
                                        π s2 +1

                            ∞                                      ∞
                              cos sx       π −x                      cos tx     π −x
                     and    ∫ s 2 + 1 ds = 2 e
                            0
                                                           (or )   ∫t
                                                                   0
                                                                      2
                                                                        +1
                                                                            dt = e
                                                                                2


FINITE FOURIER TRANSFORMS

       If f (x) is a function defined in the interval (0 , l) then the finite Fourier sine
transform of f (x) in 0 <x < l is defined as




                                                                                             22
nπx
                                        l
                  FS [ f ( x)] = ∫ f ( x). sin           dx         where ‘n’ is an integer
                                        0
                                                      l

       The inverse finite Fourier sine transform of FS [ f (x )] is f (x) and is given by
                       2 ∞                  nπx
               f ( x) = ∑ FS [ f ( x )] sin
                       l n =1                l

       The finite Fourier cosine transform of f (x ) in 0 < x < l is defined as
                                                       nπx
                                                l
                   FC [ f ( x)] = ∫ f ( x). cos            dx         where ‘n’ is an integer
                                                0
                                                        l

       The inverse finite Fourier cosine transform of FC [ f (x)] is f (x) and is given by
                       1        2 ∞                  nπx
               f ( x) = FC (0) + ∑ FC [ f ( x )] cos
                       l        l n =1                l

PROBLEMS

1. Find the finite Fourier sine and cosine transforms of f ( x) = x 2 in 0 < x < l.

Solution:
       The finite Fourier sine transform is

                                                    nπx
                                    l
                 FS [ f ( x)] = ∫ f ( x). sin           dx
                                    0
                                                     l
Here      f ( x) = x   2




                              [ ]                   nπx
                                            l
                           FS x 2 = ∫ x 2 . sin         dx
                                            0
                                                     l

                                                                                                l
                                              nπx              nπx       nπx 
                                        − cos           − sin        cos     
                                    = x 2      l  − 2 x         l  + 2    l 
                                           nπ           n 2π 2   n 3π 3 
                                                                            
                                             l               l2       l 3  0  


                                            − l3          2l 3         2l 3
                                    =            cos nπ + 3 3 cos nπ − 3 3
                                            nπ           nπ           nπ


                                    =
                                            l3
                                            nπ
                                                           2l 3
                                                                [
                                               (−1) n +1 + 3 3 (−1) n − 1
                                                          nπ
                                                                            ]
The finite Fourier cosine transform is



                                                                                                    23
nπx
                                        l
                     FC [ f ( x)] = ∫ f ( x). cos       dx
                                        0
                                                     l
Here        f ( x) = x   2




                             [ ]                 nπx
                                        l
                         FC x 2 = ∫ x 2 . cos        dx
                                        0
                                                  l
                                                                              l
                             nπx              nπx          nπx 
                         sin           − cos        − sin      
                     = x 2    l  − 2 x         l  + 2       l 
                         nπ            n 2π 2   n 3π 3 
                         l                                    
                                            l2           l3      0


                      2l 3
                 =          cos nπ
                     n 2π 2

                     2l 3
                 =        (−1) n
                     nπ
                      2 2



2. Find the finite Fourier sine and cosine transforms of f ( x ) = x in (0 , π ) .

Solution:
       The finite Fourier sine transform of f ( x) = x in (0 , π ) is
                                   π
                 FS [ f ( x)] = ∫ f ( x ). sin nx dx
                                   0

Here        f ( x) = x in (0 , π )
                                   π                                              π
                                                    − cos nx   − sin nx 
                      FS [ x ] = ∫ x. sin nx dx =  x          − 1  2    
                                 0                     n       n         0

                                       π                      π
                              =−         cos nπ = (−1) n +1 .
                                       n                      n

The finite Fourier cosine transform of f ( x) = x in (0 , π ) is
                                   π
                 FC [ f ( x)] = ∫ f ( x). cos nx dx
                                   0

Here        f ( x) = x in (0 , π )
                                   π                                         π
                                                    sin nx   − cos nx 
                      FC [ x ] = ∫ x. cos nx dx =  x        − 1  2    
                                 0                  n   n              0


                              =
                                   1
                                   n 2
                                                1   1
                                                             [
                                       cos nπ − 2 = 2 (−1) n − 1
                                               n   n
                                                                  ]


                                                                                      24
2π (−1) p−1
3. Find f (x) if its finite sine transform is given by                                          , where p is positive
                                                                                        p3
integer and 0 < x < π .

Solution:
       We know that the inverse Fourier sine transform is given by

                                       ∞
                               2
                    f ( x) =        ∑        FS [ f ( x )] sin px                                          ………………..(1)
                               π     p =1

                              2π (−1) p−1
Here     FS [ f (x )] =                                                                                    ………………..(2)
                                  p3

Substituting (2) in (1), we get
                   2 ∞ 2π (−1) p −1
           f ( x) = ∑               sin px
                   π p =1    p3

                         ∞
                             (−1) p −1
                    = 4∑               sin px
                        p =1   p3

                    2 pπ 
               cos         
                    3  find FC [ f ( p )] if 0 < x <1.
                                −1
4. If
      f ( p) =
                (2 p + 1) 2

Solution:
                                                                           ∞
                                                                                                     nπx
       We know that FC
                                     −1
                                           [ f ( p)] = 1 FC (0) + 2 ∑             FC [ f ( x)] cos
                                                         l             l   n =1                       l
                       2 pπ 
                  cos         
Here
         f ( p) =      3 
                   (2 p + 1) 2

Let    FC [ f ( x)] = f ( p)

                                                  ∞
                                                                       nπx
 ∴ FC
         −1
              [ f ( p)] = 1    f C ( 0) +
                                              2
                                                  ∑      f ( p ) cos                      [ l = 1]
                          l                   l   n =1                  l


                                                 2 pπ 
                                   ∞
                                            cos         
                      = 1 + 2∑                   3  . cos nπx
                                   n =1      (2 p + 1) 2




                                                                                                                        25
UNIT-4
                                                                         PART A

1. State the Fourier integral theorem.
Ans:
       If f (x) is a given function defined in (-l , l) and satisfies Dirichlet’s conditions, then
                                     ∞ ∞
                                 1
                  f ( x) =
                                 π   ∫ ∫ f (t ) cos λ (t − x) dt dλ
                                         0 −∞



2. State the convolution theorem of the Fourier transform.
Ans:
        If F(s) and G(s) are the Fourier transform of f (x ) and g (x) respectively then the
Fourier transform of the convolution of f(x) and g(x) is the product of their Fourier
transforms.
        i.e., F [ f ( x ) * g ( x)] = F ( s ).G ( s)

3. Write the Fourier transform pair.
Ans:
F [ f (x)] and F −1 [ F ( S )] are Fourier transform pairs.

4. Find the Fourier sine transform of f ( x) = e − ax (a > 0).
Ans:
                      ∞
                    2
     FS [ f ( x)] =
                    π ∫
                        f ( x ). sin sx dx
                      0

                      2
                           ∞
                                                                   2 s                            ∞ −ax              b 
                           ∫ e . sin sx dx =                                                        ∫ e sin bx dx = 2
                              − ax
                 =                                                                                                          
                      π      0
                                                                   π  s2 + a2 
                                                                                                  0               a + b2 
                      2 s 
                 =
                      π  s2 + a2 
                                 

5. If the Fourier transform of f (x ) is F(s) then prove that . F [ f ( x − a )] = e isa F ( s)
Ans:
                                ∞
                             1
           F [ f ( x − a)] =     ∫∞ f ( x − a).e dx
                                                isx

                             2π −
Put        x-a = y
           dx = dy
When x = −∞, y = −∞ and x = ∞, y = ∞
                                            ∞                                             ∞
                                   1                                              e ias
         F [ f ( x − a)] =                  ∫ f ( y).e
                                                          is ( y + a )
                                                                         . dy =           ∫ f ( y).e
                                                                                                       isy
                                                                                                             .dy
                                   2π       −∞                                     2π     −∞



                                            ∞
                                 e ias
                       =                    ∫ f ( x).e         .dx = e isa F ( s )
                                                         isx

                                  2π       −∞


6. State the Fourier transforms of the derivatives of a function.


                                                                                                                                26
Chapter 4 (maths 3)
Chapter 4 (maths 3)
Chapter 4 (maths 3)
Chapter 4 (maths 3)
Chapter 4 (maths 3)

Weitere ähnliche Inhalte

Was ist angesagt?

periodic functions and Fourier series
periodic functions and Fourier seriesperiodic functions and Fourier series
periodic functions and Fourier seriesUmang Gupta
 
Application of-differential-equation-in-real-life
Application of-differential-equation-in-real-lifeApplication of-differential-equation-in-real-life
Application of-differential-equation-in-real-lifeRazwanul Ghani
 
Partial differentiation
Partial differentiationPartial differentiation
Partial differentiationTanuj Parikh
 
Presentation on Solution to non linear equations
Presentation on Solution to non linear equationsPresentation on Solution to non linear equations
Presentation on Solution to non linear equationsRifat Rahamatullah
 
Fourier series and its applications by md nazmul islam
Fourier series and its applications by md nazmul islamFourier series and its applications by md nazmul islam
Fourier series and its applications by md nazmul islamMd Nazmul Islam
 
History and Real Life Applications of Fourier Analaysis
History and Real Life Applications of Fourier AnalaysisHistory and Real Life Applications of Fourier Analaysis
History and Real Life Applications of Fourier AnalaysisSyed Ahmed Zaki
 
Partial differential equations
Partial differential equationsPartial differential equations
Partial differential equationsaman1894
 
APPLICATION OF PARTIAL DIFFERENTIATION
APPLICATION OF PARTIAL DIFFERENTIATIONAPPLICATION OF PARTIAL DIFFERENTIATION
APPLICATION OF PARTIAL DIFFERENTIATIONDhrupal Patel
 
application of partial differentiation
application of partial differentiationapplication of partial differentiation
application of partial differentiationeteaching
 

Was ist angesagt? (20)

Fourier series
Fourier seriesFourier series
Fourier series
 
Laplace transform
Laplace transformLaplace transform
Laplace transform
 
periodic functions and Fourier series
periodic functions and Fourier seriesperiodic functions and Fourier series
periodic functions and Fourier series
 
Application of-differential-equation-in-real-life
Application of-differential-equation-in-real-lifeApplication of-differential-equation-in-real-life
Application of-differential-equation-in-real-life
 
Fourier series
Fourier seriesFourier series
Fourier series
 
Partial differentiation
Partial differentiationPartial differentiation
Partial differentiation
 
Presentation on Solution to non linear equations
Presentation on Solution to non linear equationsPresentation on Solution to non linear equations
Presentation on Solution to non linear equations
 
1. introduction to complex numbers
1. introduction to complex numbers1. introduction to complex numbers
1. introduction to complex numbers
 
Fourier series and its applications by md nazmul islam
Fourier series and its applications by md nazmul islamFourier series and its applications by md nazmul islam
Fourier series and its applications by md nazmul islam
 
History and Real Life Applications of Fourier Analaysis
History and Real Life Applications of Fourier AnalaysisHistory and Real Life Applications of Fourier Analaysis
History and Real Life Applications of Fourier Analaysis
 
Partial differential equations
Partial differential equationsPartial differential equations
Partial differential equations
 
Fourier integral
Fourier integralFourier integral
Fourier integral
 
Stoke’s theorem
Stoke’s theoremStoke’s theorem
Stoke’s theorem
 
1 d wave equation
1 d wave equation1 d wave equation
1 d wave equation
 
Properties of laplace transform
Properties of laplace transformProperties of laplace transform
Properties of laplace transform
 
APPLICATION OF PARTIAL DIFFERENTIATION
APPLICATION OF PARTIAL DIFFERENTIATIONAPPLICATION OF PARTIAL DIFFERENTIATION
APPLICATION OF PARTIAL DIFFERENTIATION
 
Application of derivative
Application of derivativeApplication of derivative
Application of derivative
 
Double integration
Double integrationDouble integration
Double integration
 
Fourier transform
Fourier transformFourier transform
Fourier transform
 
application of partial differentiation
application of partial differentiationapplication of partial differentiation
application of partial differentiation
 

Andere mochten auch

N.P. BALI ENGINEERING MATH REAL
N.P. BALI ENGINEERING MATH REALN.P. BALI ENGINEERING MATH REAL
N.P. BALI ENGINEERING MATH REALchucky oz
 
Numerical Solution of Nth - Order Fuzzy Initial Value Problems by Fourth Orde...
Numerical Solution of Nth - Order Fuzzy Initial Value Problems by Fourth Orde...Numerical Solution of Nth - Order Fuzzy Initial Value Problems by Fourth Orde...
Numerical Solution of Nth - Order Fuzzy Initial Value Problems by Fourth Orde...IOSR Journals
 
Free Ebooks Download ! Edhole
Free Ebooks Download ! EdholeFree Ebooks Download ! Edhole
Free Ebooks Download ! EdholeEdhole.com
 
Free Download Powerpoint Slides
Free Download Powerpoint SlidesFree Download Powerpoint Slides
Free Download Powerpoint SlidesGeorge
 

Andere mochten auch (9)

Chapter 2 (maths 3)
Chapter 2 (maths 3)Chapter 2 (maths 3)
Chapter 2 (maths 3)
 
Chapter 3 (maths 3)
Chapter 3 (maths 3)Chapter 3 (maths 3)
Chapter 3 (maths 3)
 
Chapter 1 (maths 3)
Chapter 1 (maths 3)Chapter 1 (maths 3)
Chapter 1 (maths 3)
 
Chapter 5 (maths 3)
Chapter 5 (maths 3)Chapter 5 (maths 3)
Chapter 5 (maths 3)
 
mathematics formulas
mathematics formulasmathematics formulas
mathematics formulas
 
N.P. BALI ENGINEERING MATH REAL
N.P. BALI ENGINEERING MATH REALN.P. BALI ENGINEERING MATH REAL
N.P. BALI ENGINEERING MATH REAL
 
Numerical Solution of Nth - Order Fuzzy Initial Value Problems by Fourth Orde...
Numerical Solution of Nth - Order Fuzzy Initial Value Problems by Fourth Orde...Numerical Solution of Nth - Order Fuzzy Initial Value Problems by Fourth Orde...
Numerical Solution of Nth - Order Fuzzy Initial Value Problems by Fourth Orde...
 
Free Ebooks Download ! Edhole
Free Ebooks Download ! EdholeFree Ebooks Download ! Edhole
Free Ebooks Download ! Edhole
 
Free Download Powerpoint Slides
Free Download Powerpoint SlidesFree Download Powerpoint Slides
Free Download Powerpoint Slides
 

Ähnlich wie Chapter 4 (maths 3)

Lesson 14: Derivatives of Logarithmic and Exponential Functions (slides)
Lesson 14: Derivatives of Logarithmic and Exponential Functions (slides)Lesson 14: Derivatives of Logarithmic and Exponential Functions (slides)
Lesson 14: Derivatives of Logarithmic and Exponential Functions (slides)Matthew Leingang
 
Lesson 14: Derivatives of Logarithmic and Exponential Functions (slides)
Lesson 14: Derivatives of Logarithmic and Exponential Functions (slides)Lesson 14: Derivatives of Logarithmic and Exponential Functions (slides)
Lesson 14: Derivatives of Logarithmic and Exponential Functions (slides)Mel Anthony Pepito
 
SOME THOUGHTS ON DIVERGENT SERIES
SOME THOUGHTS ON DIVERGENT SERIESSOME THOUGHTS ON DIVERGENT SERIES
SOME THOUGHTS ON DIVERGENT SERIESgenius98
 
some thoughts on divergent series
some thoughts on divergent seriessome thoughts on divergent series
some thoughts on divergent seriesgenius98
 
Varian, microeconomic analysis, solution book
Varian, microeconomic analysis, solution bookVarian, microeconomic analysis, solution book
Varian, microeconomic analysis, solution bookJosé Antonio PAYANO YALE
 
Lesson 28: The Fundamental Theorem of Calculus
Lesson 28: The Fundamental Theorem of CalculusLesson 28: The Fundamental Theorem of Calculus
Lesson 28: The Fundamental Theorem of CalculusMatthew Leingang
 
Lesson 28: The Fundamental Theorem of Calculus
Lesson 28: The Fundamental Theorem of CalculusLesson 28: The Fundamental Theorem of Calculus
Lesson 28: The Fundamental Theorem of CalculusMatthew Leingang
 
Lattices and codes
Lattices and codesLattices and codes
Lattices and codesSpringer
 
Introduction to inverse problems
Introduction to inverse problemsIntroduction to inverse problems
Introduction to inverse problemsDelta Pi Systems
 
Lesson 26: The Fundamental Theorem of Calculus (slides)
Lesson 26: The Fundamental Theorem of Calculus (slides)Lesson 26: The Fundamental Theorem of Calculus (slides)
Lesson 26: The Fundamental Theorem of Calculus (slides)Matthew Leingang
 
Lesson 26: The Fundamental Theorem of Calculus (slides)
Lesson 26: The Fundamental Theorem of Calculus (slides)Lesson 26: The Fundamental Theorem of Calculus (slides)
Lesson 26: The Fundamental Theorem of Calculus (slides)Mel Anthony Pepito
 
Lesson 26: The Fundamental Theorem of Calculus (slides)
Lesson 26: The Fundamental Theorem of Calculus (slides)Lesson 26: The Fundamental Theorem of Calculus (slides)
Lesson 26: The Fundamental Theorem of Calculus (slides)Matthew Leingang
 
IJCER (www.ijceronline.com) International Journal of computational Engineerin...
IJCER (www.ijceronline.com) International Journal of computational Engineerin...IJCER (www.ijceronline.com) International Journal of computational Engineerin...
IJCER (www.ijceronline.com) International Journal of computational Engineerin...ijceronline
 

Ähnlich wie Chapter 4 (maths 3) (20)

Fourier series
Fourier seriesFourier series
Fourier series
 
Lesson 14: Derivatives of Logarithmic and Exponential Functions (slides)
Lesson 14: Derivatives of Logarithmic and Exponential Functions (slides)Lesson 14: Derivatives of Logarithmic and Exponential Functions (slides)
Lesson 14: Derivatives of Logarithmic and Exponential Functions (slides)
 
Lesson 14: Derivatives of Logarithmic and Exponential Functions (slides)
Lesson 14: Derivatives of Logarithmic and Exponential Functions (slides)Lesson 14: Derivatives of Logarithmic and Exponential Functions (slides)
Lesson 14: Derivatives of Logarithmic and Exponential Functions (slides)
 
SOME THOUGHTS ON DIVERGENT SERIES
SOME THOUGHTS ON DIVERGENT SERIESSOME THOUGHTS ON DIVERGENT SERIES
SOME THOUGHTS ON DIVERGENT SERIES
 
some thoughts on divergent series
some thoughts on divergent seriessome thoughts on divergent series
some thoughts on divergent series
 
Ism et chapter_3
Ism et chapter_3Ism et chapter_3
Ism et chapter_3
 
Ism et chapter_3
Ism et chapter_3Ism et chapter_3
Ism et chapter_3
 
Ism et chapter_3
Ism et chapter_3Ism et chapter_3
Ism et chapter_3
 
Funcion gamma
Funcion gammaFuncion gamma
Funcion gamma
 
Taylor problem
Taylor problemTaylor problem
Taylor problem
 
Varian, microeconomic analysis, solution book
Varian, microeconomic analysis, solution bookVarian, microeconomic analysis, solution book
Varian, microeconomic analysis, solution book
 
Fourier series
Fourier seriesFourier series
Fourier series
 
Lesson 28: The Fundamental Theorem of Calculus
Lesson 28: The Fundamental Theorem of CalculusLesson 28: The Fundamental Theorem of Calculus
Lesson 28: The Fundamental Theorem of Calculus
 
Lesson 28: The Fundamental Theorem of Calculus
Lesson 28: The Fundamental Theorem of CalculusLesson 28: The Fundamental Theorem of Calculus
Lesson 28: The Fundamental Theorem of Calculus
 
Lattices and codes
Lattices and codesLattices and codes
Lattices and codes
 
Introduction to inverse problems
Introduction to inverse problemsIntroduction to inverse problems
Introduction to inverse problems
 
Lesson 26: The Fundamental Theorem of Calculus (slides)
Lesson 26: The Fundamental Theorem of Calculus (slides)Lesson 26: The Fundamental Theorem of Calculus (slides)
Lesson 26: The Fundamental Theorem of Calculus (slides)
 
Lesson 26: The Fundamental Theorem of Calculus (slides)
Lesson 26: The Fundamental Theorem of Calculus (slides)Lesson 26: The Fundamental Theorem of Calculus (slides)
Lesson 26: The Fundamental Theorem of Calculus (slides)
 
Lesson 26: The Fundamental Theorem of Calculus (slides)
Lesson 26: The Fundamental Theorem of Calculus (slides)Lesson 26: The Fundamental Theorem of Calculus (slides)
Lesson 26: The Fundamental Theorem of Calculus (slides)
 
IJCER (www.ijceronline.com) International Journal of computational Engineerin...
IJCER (www.ijceronline.com) International Journal of computational Engineerin...IJCER (www.ijceronline.com) International Journal of computational Engineerin...
IJCER (www.ijceronline.com) International Journal of computational Engineerin...
 

Kürzlich hochgeladen

Sociology 101 Demonstration of Learning Exhibit
Sociology 101 Demonstration of Learning ExhibitSociology 101 Demonstration of Learning Exhibit
Sociology 101 Demonstration of Learning Exhibitjbellavia9
 
1029-Danh muc Sach Giao Khoa khoi 6.pdf
1029-Danh muc Sach Giao Khoa khoi  6.pdf1029-Danh muc Sach Giao Khoa khoi  6.pdf
1029-Danh muc Sach Giao Khoa khoi 6.pdfQucHHunhnh
 
SKILL OF INTRODUCING THE LESSON MICRO SKILLS.pptx
SKILL OF INTRODUCING THE LESSON MICRO SKILLS.pptxSKILL OF INTRODUCING THE LESSON MICRO SKILLS.pptx
SKILL OF INTRODUCING THE LESSON MICRO SKILLS.pptxAmanpreet Kaur
 
Basic Civil Engineering first year Notes- Chapter 4 Building.pptx
Basic Civil Engineering first year Notes- Chapter 4 Building.pptxBasic Civil Engineering first year Notes- Chapter 4 Building.pptx
Basic Civil Engineering first year Notes- Chapter 4 Building.pptxDenish Jangid
 
ICT Role in 21st Century Education & its Challenges.pptx
ICT Role in 21st Century Education & its Challenges.pptxICT Role in 21st Century Education & its Challenges.pptx
ICT Role in 21st Century Education & its Challenges.pptxAreebaZafar22
 
TỔNG ÔN TẬP THI VÀO LỚP 10 MÔN TIẾNG ANH NĂM HỌC 2023 - 2024 CÓ ĐÁP ÁN (NGỮ Â...
TỔNG ÔN TẬP THI VÀO LỚP 10 MÔN TIẾNG ANH NĂM HỌC 2023 - 2024 CÓ ĐÁP ÁN (NGỮ Â...TỔNG ÔN TẬP THI VÀO LỚP 10 MÔN TIẾNG ANH NĂM HỌC 2023 - 2024 CÓ ĐÁP ÁN (NGỮ Â...
TỔNG ÔN TẬP THI VÀO LỚP 10 MÔN TIẾNG ANH NĂM HỌC 2023 - 2024 CÓ ĐÁP ÁN (NGỮ Â...Nguyen Thanh Tu Collection
 
ICT role in 21st century education and it's challenges.
ICT role in 21st century education and it's challenges.ICT role in 21st century education and it's challenges.
ICT role in 21st century education and it's challenges.MaryamAhmad92
 
Salient Features of India constitution especially power and functions
Salient Features of India constitution especially power and functionsSalient Features of India constitution especially power and functions
Salient Features of India constitution especially power and functionsKarakKing
 
HMCS Max Bernays Pre-Deployment Brief (May 2024).pptx
HMCS Max Bernays Pre-Deployment Brief (May 2024).pptxHMCS Max Bernays Pre-Deployment Brief (May 2024).pptx
HMCS Max Bernays Pre-Deployment Brief (May 2024).pptxEsquimalt MFRC
 
Key note speaker Neum_Admir Softic_ENG.pdf
Key note speaker Neum_Admir Softic_ENG.pdfKey note speaker Neum_Admir Softic_ENG.pdf
Key note speaker Neum_Admir Softic_ENG.pdfAdmir Softic
 
Application orientated numerical on hev.ppt
Application orientated numerical on hev.pptApplication orientated numerical on hev.ppt
Application orientated numerical on hev.pptRamjanShidvankar
 
Activity 01 - Artificial Culture (1).pdf
Activity 01 - Artificial Culture (1).pdfActivity 01 - Artificial Culture (1).pdf
Activity 01 - Artificial Culture (1).pdfciinovamais
 
Holdier Curriculum Vitae (April 2024).pdf
Holdier Curriculum Vitae (April 2024).pdfHoldier Curriculum Vitae (April 2024).pdf
Holdier Curriculum Vitae (April 2024).pdfagholdier
 
Towards a code of practice for AI in AT.pptx
Towards a code of practice for AI in AT.pptxTowards a code of practice for AI in AT.pptx
Towards a code of practice for AI in AT.pptxJisc
 
Kodo Millet PPT made by Ghanshyam bairwa college of Agriculture kumher bhara...
Kodo Millet  PPT made by Ghanshyam bairwa college of Agriculture kumher bhara...Kodo Millet  PPT made by Ghanshyam bairwa college of Agriculture kumher bhara...
Kodo Millet PPT made by Ghanshyam bairwa college of Agriculture kumher bhara...pradhanghanshyam7136
 
Dyslexia AI Workshop for Slideshare.pptx
Dyslexia AI Workshop for Slideshare.pptxDyslexia AI Workshop for Slideshare.pptx
Dyslexia AI Workshop for Slideshare.pptxcallscotland1987
 
How to Create and Manage Wizard in Odoo 17
How to Create and Manage Wizard in Odoo 17How to Create and Manage Wizard in Odoo 17
How to Create and Manage Wizard in Odoo 17Celine George
 
On National Teacher Day, meet the 2024-25 Kenan Fellows
On National Teacher Day, meet the 2024-25 Kenan FellowsOn National Teacher Day, meet the 2024-25 Kenan Fellows
On National Teacher Day, meet the 2024-25 Kenan FellowsMebane Rash
 
Food safety_Challenges food safety laboratories_.pdf
Food safety_Challenges food safety laboratories_.pdfFood safety_Challenges food safety laboratories_.pdf
Food safety_Challenges food safety laboratories_.pdfSherif Taha
 

Kürzlich hochgeladen (20)

Sociology 101 Demonstration of Learning Exhibit
Sociology 101 Demonstration of Learning ExhibitSociology 101 Demonstration of Learning Exhibit
Sociology 101 Demonstration of Learning Exhibit
 
1029-Danh muc Sach Giao Khoa khoi 6.pdf
1029-Danh muc Sach Giao Khoa khoi  6.pdf1029-Danh muc Sach Giao Khoa khoi  6.pdf
1029-Danh muc Sach Giao Khoa khoi 6.pdf
 
SKILL OF INTRODUCING THE LESSON MICRO SKILLS.pptx
SKILL OF INTRODUCING THE LESSON MICRO SKILLS.pptxSKILL OF INTRODUCING THE LESSON MICRO SKILLS.pptx
SKILL OF INTRODUCING THE LESSON MICRO SKILLS.pptx
 
Basic Civil Engineering first year Notes- Chapter 4 Building.pptx
Basic Civil Engineering first year Notes- Chapter 4 Building.pptxBasic Civil Engineering first year Notes- Chapter 4 Building.pptx
Basic Civil Engineering first year Notes- Chapter 4 Building.pptx
 
ICT Role in 21st Century Education & its Challenges.pptx
ICT Role in 21st Century Education & its Challenges.pptxICT Role in 21st Century Education & its Challenges.pptx
ICT Role in 21st Century Education & its Challenges.pptx
 
TỔNG ÔN TẬP THI VÀO LỚP 10 MÔN TIẾNG ANH NĂM HỌC 2023 - 2024 CÓ ĐÁP ÁN (NGỮ Â...
TỔNG ÔN TẬP THI VÀO LỚP 10 MÔN TIẾNG ANH NĂM HỌC 2023 - 2024 CÓ ĐÁP ÁN (NGỮ Â...TỔNG ÔN TẬP THI VÀO LỚP 10 MÔN TIẾNG ANH NĂM HỌC 2023 - 2024 CÓ ĐÁP ÁN (NGỮ Â...
TỔNG ÔN TẬP THI VÀO LỚP 10 MÔN TIẾNG ANH NĂM HỌC 2023 - 2024 CÓ ĐÁP ÁN (NGỮ Â...
 
ICT role in 21st century education and it's challenges.
ICT role in 21st century education and it's challenges.ICT role in 21st century education and it's challenges.
ICT role in 21st century education and it's challenges.
 
Salient Features of India constitution especially power and functions
Salient Features of India constitution especially power and functionsSalient Features of India constitution especially power and functions
Salient Features of India constitution especially power and functions
 
HMCS Max Bernays Pre-Deployment Brief (May 2024).pptx
HMCS Max Bernays Pre-Deployment Brief (May 2024).pptxHMCS Max Bernays Pre-Deployment Brief (May 2024).pptx
HMCS Max Bernays Pre-Deployment Brief (May 2024).pptx
 
Key note speaker Neum_Admir Softic_ENG.pdf
Key note speaker Neum_Admir Softic_ENG.pdfKey note speaker Neum_Admir Softic_ENG.pdf
Key note speaker Neum_Admir Softic_ENG.pdf
 
Application orientated numerical on hev.ppt
Application orientated numerical on hev.pptApplication orientated numerical on hev.ppt
Application orientated numerical on hev.ppt
 
Spatium Project Simulation student brief
Spatium Project Simulation student briefSpatium Project Simulation student brief
Spatium Project Simulation student brief
 
Activity 01 - Artificial Culture (1).pdf
Activity 01 - Artificial Culture (1).pdfActivity 01 - Artificial Culture (1).pdf
Activity 01 - Artificial Culture (1).pdf
 
Holdier Curriculum Vitae (April 2024).pdf
Holdier Curriculum Vitae (April 2024).pdfHoldier Curriculum Vitae (April 2024).pdf
Holdier Curriculum Vitae (April 2024).pdf
 
Towards a code of practice for AI in AT.pptx
Towards a code of practice for AI in AT.pptxTowards a code of practice for AI in AT.pptx
Towards a code of practice for AI in AT.pptx
 
Kodo Millet PPT made by Ghanshyam bairwa college of Agriculture kumher bhara...
Kodo Millet  PPT made by Ghanshyam bairwa college of Agriculture kumher bhara...Kodo Millet  PPT made by Ghanshyam bairwa college of Agriculture kumher bhara...
Kodo Millet PPT made by Ghanshyam bairwa college of Agriculture kumher bhara...
 
Dyslexia AI Workshop for Slideshare.pptx
Dyslexia AI Workshop for Slideshare.pptxDyslexia AI Workshop for Slideshare.pptx
Dyslexia AI Workshop for Slideshare.pptx
 
How to Create and Manage Wizard in Odoo 17
How to Create and Manage Wizard in Odoo 17How to Create and Manage Wizard in Odoo 17
How to Create and Manage Wizard in Odoo 17
 
On National Teacher Day, meet the 2024-25 Kenan Fellows
On National Teacher Day, meet the 2024-25 Kenan FellowsOn National Teacher Day, meet the 2024-25 Kenan Fellows
On National Teacher Day, meet the 2024-25 Kenan Fellows
 
Food safety_Challenges food safety laboratories_.pdf
Food safety_Challenges food safety laboratories_.pdfFood safety_Challenges food safety laboratories_.pdf
Food safety_Challenges food safety laboratories_.pdf
 

Chapter 4 (maths 3)

  • 1. CHAPTER 4 FOURIER TRANSFORMS INTEGRAL TRANSFORM b The integral transform of a function f (x ) is defined by ∫ f ( x).k (s , x)dx where a k(s , x) is a known function of s and x and it is called the kernel of the transform. When k(s , x) is a sine or cosine function, we get transforms called Fourier sine or cosine transforms. FOURIER INTEGRAL THEOREM If f (x) is a given function defined in (-l , l) and satisfies Dirichlet’s conditions, then ∞ ∞ 1 π ∫ −∫ f ( x) = f (t ) cos λ (t − x) dt dλ 0 ∞ At a point of discontinuity the value of the integral on the left of above equation is 1 { f ( x + 0) − f ( x − 0)}. 2 EXAMPLES 1 for x ≤ 1  1. Express the function f ( x) =  as a Fourier Integral. Hence evaluate 0 for x > 1  ∞ ∞ sin λ cos λx sin λ ∫ λ 0 dλ and find the value of ∫ 0 λ dλ . Solution: We know that the Fourier Integral formula for f (x) is ∞ ∞ 1 f ( x) = π ∫ ∫ f (t ) cos λ (t − x) dt dλ 0 −∞ ……………….(1) Here f (t ) = 1 for t ≤ 1 i.e., f(t) = 1 in -1 < t < 1 f (t ) = 0 for t > 1 f (t ) = 0 in − ∞ < t < −1 and 1 < t < ∞ ∞ 1 1 ∴ Equation (1) ⇒ f ( x) = π ∫ ∫ cos λ (t − x) dt dλ 0 −1 ∞ 1 1  sin λ (t − x)  π ∫ =   dλ 0 λ  −1 ∞ 1 sin λ (1 − x) − sin λ (−1 − x) π∫ = dλ 0 λ ∞ 1 sin λ (1 − x) + sin λ (1 + x ) = ∫ dλ π 0 λ 1
  • 2. 2 sin λ cos λx π∫ ∴ f ( x) = dλ .………………(2) 0 λ [Using sin (A+B) + sin (A-B) = 2 sin A cos B] This is Fourier Integral of the given function. From (2) we get ∞ sin λ cos λx π ∫ 0 λ dλ = 2 f ( x) ……………….(3  1 for x ≤ 1 But f ( x) =  ………………..(4) 0 for x > 1  Substituting (4) in (3) we get π ∞ sin λ cos λx  for x ≤ 1 ∫ λ dλ =  2 0 0 for x > 1  ∞ sin λ π Putting x = 0 we get ∫0 λ dλ = 2 2. Find the Fourier Integral of the function 0 x<0 1  f ( x) =  x=0 2 e − x x > 0  Verify the representation directly at the point x = 0. Solution: The Fourier integral of f (x) is ∞ ∞ 1 f ( x) = π ∫ ∫ f (t ) cos λ (t − x) dt dλ 0 −∞ ……………….(1) 1   ∞ 0 ∞ = ∫ −∫∞ π 0 f (t ) cos λ (t − x )dt + ∫ f (t ) cos λ (t − x)dt d λ 0  1   ∞ 0 ∞ = ∫  ∫ 0. cos λ (t − x)dt + ∫ e −t cos λ (t − x)dt dλ π 0 − ∞ 0  ∞ ∞ 1  e −t = ∫ 2 [ − cos( λt − λx ) + λ sin(λt − λx)]  dλ  π 0 λ +1 0 ∞ 1 cos λx + λ sin λx π∫ f (x) = dλ ……….………(2) 0 λ2 + 1 2
  • 3. Putting x = 0 in (2), we get ∞ 1 f (0) = ∫ 2 1 π 0 λ +1 1 d λ = tan −1 ( λ ) 0 π ∞ [ ] 1 [ = tan −1 ( ∞ ) − tan −1 (0) π ] 1 π  1 =  = π 2 2 1 The value of the given function at x = 0 is . Hence verified. 2 FOURIER SINE AND COSINE INTEGRALS The integral of the form 2∞ ∞ f ( x) = ∫ sin λx ∫ f (t ) sin λt dt dλ π0 0 is known as Fourier sine integral. The integral of the form ∞ ∞ 2 f ( x) = ∫ cos λx ∫ f (t ) cos λt dt dλ π 0 0 is known as Fourier cosine integral. PROBLEMS 1. Using Fourier integral formula, prove that 2(b 2 − a 2 ) ∞ u sin xu ∫ (u 2 + a 2 )(u 2 + b 2 ) du (a, b > 0) − ax − bx e −e = π 0 Solution: The presence of sin xu in the integral suggests that the Fourier sine integral formula has been used. Fourier sine integral representation is given by ∞ ∞ 2 f ( x) = ∫ sin ux ∫ f (t ) sin ut dt du π 0 0 ∞ ∞  sin ux du  ∫ ( e − at − e −bt ) sin ut dt  2 e − ax − e −bx = ∫ π 0 0  3
  • 4. ∞ 2  e − at − bt  = ∫ sin ux du  2 { − a sin ut − u cos ut} − 2e 2 { − b sin ut − u cos ut}  π 0 a + u 2 b +u 0 ∞ 2  u u  = ∫ sin ux du  a 2 + u 2 − b 2 + u 2  π 0   ∞ 2(b 2 − a 2 ) u sin ux = π ∫ (u 2 + a 2 )(u 2 + b 2 ) du 0 2. Using Fourier integral formula, prove that 2 ∞ (λ 2 + 2 ) cos xλ ∫ dλ −x e cos x = π 0 λ2 + 4 Solution: The presence of cos xλ in the integral suggests that the Fourier cosine integral formula for e − x cos x has been used. Fourier cosine integral representation is given by ∞ ∞ 2 π∫ f ( x) = cos λx ∫ f (t ) cos λt dt dλ 0 0 2∞ ∞ − t  ∴e −x cos x = ∫ cos xλ dλ  ∫ e cos t cos λt dt  π 0 0  2 ∞ 1 ∞  = ∫ cos xλ dλ  ∫ e −t { cos(λ + 1)t + cos(λ − 1)t } dt  π 0 2 0  ∞  = 2 π 0  1 −t [ ∫ cos xλ dλ  (λ + 1) 2 + 1 e { − cos(λ + 1)t + (λ + 1) sin(λ + 1)t} ] ∞ 0  + 1 (λ − 1) + 1 2 [ e −t { − cos(λ − 1)t + (λ − 1) sin(λ − 1)t } 0 ∞ ] ) ∞ 1  1 1  = ∫ +  cos xλ dλ π 0  (λ + 1) + 1 (λ − 1) + 1 2 2 ∞ 2 (λ2 + 2) cos xλ = ∫ dλ. π 0 λ2 + 4 COMPLEX FORM OF FOURIER INTEGRALS 4
  • 5. The integral of the form ∞ ∞ 1 ∫ e − iλx ∫ f (t ) e iλt f ( x) = dt d λ 2π −∞ −∞ is known as Complex form of Fourier Integral. FOURIER TRANSFORMS COMPLEX FOURIER TRANSFORMS ∞ 1 The function F [ f ( x)] = ∫∞ f (t ).e dt is called the Complex Fourier transform ist 2π − of f (x ) . INVERSION FORMULA FOR THE COMPLEX FOURIER TRANSFORM ∞ 1 ∫∞F [ f ( x)].e ds is called the inversion formula for the −isx The function f ( x) = 2π − Complex Fourier transform of F [ f ( x)] and it is denoted by F −1 [ F ( f ( x))]. FOURIER SINE TRANSFORMS ∞ 2 The function FS [ f ( x )] = ∫ f (t ).sin st dt is called the Fourier Sine Transform of π 0 the function f (x ) . ∞ 2 The function f ( x) = π ∫ F [ f ( x)]. sin sx ds is called the inversion formula for the 0 S Fourier sine transform and it is denoted by FS −1 [ FS ( f ( x))]. FOURIER COSINE TRANSFORMS ∞ 2 The function FC [ f ( x)] = π ∫ f (t ). cos st dt is called the Fourier Cosine 0 Transform of f (x) . ∞ 2 The function f ( x) = π ∫ F [ f ( x)]. cos sx ds 0 C is called the inversion formula for the Fourier Cosine Transform and it is denoted by FC −1 [ FC ( f ( x))]. PROBLEMS 1. Find the Fourier Transform of 1 − x 2 in x ≤ 1  f ( x) =  0  in x > 1 5
  • 6. sin s − s cos s s 3π Hence prove that ∫ 0 s 3 cos ds = 2 16 . Solution: We know that the Fourier transform of f (x) is given by ∞ 1 F [ f ( x )] = ∫ f ( x).e isx dx 2π −∞ −1 1 ∞ 1 1 1 = ∫ f ( x).e dx + ∫ f ( x).e dx + ∫ f ( x).e isx isx isx dx 2π −∞ 2π −1 2π 1 −1 1 ∞ 1 1 1 = ∫ 0.e dx + ∫ (1 − x ).e dx + ∫ 0.e isx 2 isx isx dx 2π −∞ 2π −1 2π 1 1 1 = ∫ (1 − x 2 ).e isx dx 2π −1 1 1  e isx e isx e isx  =  (1 − x 2 ) − ( −2 x ) 2 2 − 2 3 3  2π  is i s i s  −1 1  − 2 is 2 is − 2 −is 2 e −is  =  2 e + 3e + 2 e −  2π  s is s i s3  1  − 2 is 2  = s (e + e −is ) + 3 (e is − e −is ) 2π  2 is  1 − 4 4  1 4  =  s 2 cos s + s 3 sin s  =  s 3 (sin s − s cos s ) 2π   2π   By using inverse Fourier Transform we get ∞ 1 1 4 f ( x) = 2π . 2π ∫ −∞ s 3 (sin s − s cos s ).e −isx ds ∞ 1 4 = 2π ∫s −∞ 3 (sin s − s cos s ).(cos sx − i sin sx ) ds ∞ 1 4 = 2π ∫ −∞ s3 (sin s − s cos s ) cos sx ds ∞ 1 4 − 2π ∫ −∞ s3 (sin s − s cos s ) i sin sx ds 6
  • 7. The second integral is odd and hence its values is zero. ∞ 2 sin s − s cos s π −∫ ∴ f ( x) = cos sx ds ∞ s3 ∞ 4 sin s − s cos s = ∫ cos sx ds π 0 s3 ∞ sin s − s cos s π i.e., ∫ 0 s 3 cos sx ds = f ( x) 4 1 Putting x = , we get 2 ∞ sin s − s cos s s π  1  π  1  3π ∫0 s 3 cos ds = 2 f   = 1 −  = 4  2  4  4  16 . ∞ sin s − s cos s s 3π ∫ 0 s 3 cos ds = 2 16 . 2. Find the Fourier sine transform of e − x , x ≥ 0 (or) e − x , x > 0. Hence evaluate ∞ x sin mx ∫ 1 + x 2 dx. 0 Solution: The Fourier sine transform of f(x) is given by ∞ 2 FS [ f ( x )] = ∫ f ( x).sin sx dx π 0 −x −x Here e =e for x > 0 ∞ [ ] FS e − x = 2 π ∫e −x . sin sx dx 0 2 s ∞ − ax b  =  ∫ e sin bx dx = 2  π s2 +1 0 a + b2  Using inverse Fourier sine transform we get ∞ ∫ F [e ]. sin sx ds 2 −x f ( x) = s π 0 ∞ 2 2 s = π ∫ 0 . 2 π s +1 . sin sx ds ∞ 2 s = ∫ s 2 + 1 sin sx ds π 0 7
  • 8. π s i.e., f ( x) = ∫ 2 . sin sx ds 2 0 s +1 ∞ s. sin sx π i.e., ∫0 s +1 2 ds = e − x 2 Replacing x by m we get ∞ s. sin ms π i.e., ∫ 0 s +1 2 ds = e − m 2 ∞ x. sin mx π i.e., ∫ 0 x +1 2 dx = e −m 2 [since s is dummy variable, we can replace it by x] e − ax 3. Find the Fourier cosine transform of . x Solution: ∞ 2 We know that FC [ f ( x )] = ∫ f ( x). cos sx dx π 0 − ax e Here f ( x) = . x ∞ 2 e − ax ∴ FC [ f ( x )] = ∫ x . cos sx dx π 0 Let FC [ f ( x)] = F ( s ) ∞ 2 e − ax Then F (s) = π ∫ x . cos sx dx 0 ………………(1) Differentiating on both sides w.r.t. ‘s’ we get, ∞ dF ( s ) d 2 e − ax ds = ds π ∫ x . cos sx dx 0 ∞ 2 ∂  e −ax  = ∫ ∂s  x . cos sxdx π0   ∞ ∞ 2 e −ax 2 − ax = ∫ x (− sin sx).x dx = − π ∫ e sin sx dx π0 0 dF ( s ) 2 s  ∞ b  ∫e − ax =− . 2  sin bx dx =  ds π a + s2  0 a + b2  2 Integrating w.r.t. ‘s’ we get 8
  • 9. 2 s F (s) = − .∫ 2 ds π s + a2 2 1 1 =− . . log ( s 2 + a 2 ) = − . log ( s 2 + a 2 ) π 2 2π e − ax − e −bx 4. Find the Fourier cosine transform of . x Solution: We know that the Fourier cosine transform of f(x) is ∞ 2 FC [ f ( x )] = ∫ f ( x). cos sx dx π 0 − ax e − e −bx Here f ( x) = x ∞  e − ax − e −bx  2 e − ax − e − bx ∴ FC   x =  π ∫ x . cos sx dx 0 ∞ ∞ 2 e − ax 2 e −bx = π ∫ x . cos sx dx − π ∫ x cos sx dx 0 0  e −ax   e −bx  = Fc   − Fc    x   x  1 1 =− log ( s 2 + a 2 ) + log ( s 2 + b 2 ) 2π 2π 1  s2 + b2  = log  2  s + a2  2π   e − as 5. Find f (x) , if its sine transform is . Hence deduce that the inverse sine s 1 transform of . s Solution: We know that the inverse Fourier sine transform of FS [ f (x )] is given by ∞ 2 f ( x) = π ∫ F [ f ( x)]. sin sx ds 0 S e − as Here FS [ f ( x )] = s 9
  • 10. 2 e − as ∴ f ( x) = π ∫ s . sin sx ds 0 Differentiating w.r.t. ‘x’ on both sides, we get, d [ f ( x )] ∞ 2 e − as ∂ dx = π ∫ s . ∂x (sin sx) ds 0 ∞ ∞ 2 e − as 2 − as = π ∫ s . cos sx s ds = π ∫ e . cos sx ds 0 0 2 a  ∞ −ax a  =  ∫ e cos bx dx = 2  π a + x2 2  0 a + b2  d [ f ( x )] 2 a = dx π x + a2 2 2 1 2 1 −1  x  ∴ f ( x) = a π ∫ x 2 + a 2 dx = a π a tan  a    2  x ∴ f ( x) = tan −1   π a 1 To find the inverse Fourier sine transform of : s Put a = 0, in (1), we get 2 2 π π f ( x) = tan −1 (∞) = . = π π 2 2 PROPERTIES 1. Linearity Property If F(s) and G(s) are the Fourier transform of f (x ) and g (x) respectively then F [ a f ( x ) + b g ( x)] = a F ( s) + b G ( s ) Proof: ∞ 1 F [ a f ( x) + b g ( x)] = ∫∞[ a f ( x) + b g ( x)] e dx isx 2π − ∞ ∞ 1 1 = ∫ a f ( x).e dx + ∫ b g ( x).e isx isx dx 2π −∞ 2π −∞ ∞ ∞ a b = ∫ f ( x).e isx dx + ∫ g ( x).e isx dx 2π −∞ 2π −∞ = a F ( s ) + b G ( s) 10
  • 11. 2. Change of Scale Property 1 s If F(s) is the Fourier transform of f (x ) then F [ f (ax)] = F  ,a>0 a a Proof: ∞ 1 F [ f (ax)] = ∫ f (ax).e isx dx 2π −∞ Put ax = y dy a dx = dy i.e., dx = a When x = −∞, y = −∞ and x = ∞, y=∞ ∞ y ∞ s 1 dy 1 1 i  y F [ f (ax)] = is 2π ∫ f ( y).e −∞ a . = a a 2π ∫ f ( y).e −∞ a .dy 1 = F ( s a) a 3. Shifting Property ( Shifting in x ) If F(s) is the Fourier transform of f (x ) then F [ f ( x − a )] = e ias F ( s ) Proof: ∞ 1 F [ f ( x − a)] = ∫ f ( x − a).e isx dx 2π −∞ Put x-a = y dx = dy When x = −∞, y = −∞ and x = ∞, y=∞ ∞ ∞ 1 e ias F [ f ( x − a)] = ∫ f ( y ).e is ( y + a ) . dy = ∫ f ( y).e isy .dy 2π −∞ 2π −∞ ∞ e ias = ∫ f ( x).e .dx = e isa F ( s ) isx 2π −∞ 4. Shifting in respect of s If F(s) is the Fourier transform of f (x ) then F e iax f ( x) = F ( s + a ) [ ] Proof: ∞ Fe[ iax ] f ( x) = 1 ∫e iax f ( x) e isx dx 2π −∞ 11
  • 12. 1 ∫ f ( x).e i( s+a) x = dx = F ( s + a) 2π −∞ 5. Modulation Theorem 1 If F(s) is the Fourier transform of f (x ) then F [ f ( x) cos ax ] = [ F ( s + a ) + F ( s − a )] 2 Proof: ∞ 1 F [ f ( x) cos ax ] = ∫ f ( x). cos ax.e isx dx 2π −∞ ∞ 1  e iax + e −iax  = ∫ 2π −∞ f ( x).e isx    2 dx   ∞ ∞ 1 1 1 1 ∫∞ f ( x).e dx + 2 . 2π ∫ f ( x).e i( s+a ) x i ( s −a ) x = . dx 2 2π − −∞ 1 1 1 = f ( s + a ) + f ( s − a ) = [ f ( s + a ) + f ( s − a )] 2 2 2 1 F [ f ( x) cos ax ] = [ F ( s + a ) + F ( s − a)] 2 COROLLARIES 1 (i ) FC [ f ( x) cos ax ] = [ FC ( s + a) + FC ( s − a)] 2 1 (ii ) FC [ f ( x) sin ax ] = [ FS (a + s) + FS (a − s )] 2 1 (iii ) FS [ f ( x) cos ax ] = [ FS ( s + a ) + FS ( s − a )] 2 1 (iv ) FS [ f ( x) sin ax ] = [ FC ( s − a ) − FC ( s + a)] 2 6. Conjugate Symmetry Property If F(s) is the Fourier transform of f (x ) then F f ( − x) = F ( s) [ ] Proof: ∞ 1 We know that F ( s ) = ∫ f ( x). e isx dx 2π −∞ Taking complex conjugate on both sides we get ∞ 1 ∫∞ f ( x). e dx −isx F (s) = 2π − 12
  • 13. Put x = -y dx = -dy When x = −∞, y = ∞ and x = ∞, y = −∞ −∞ 1 ∴ F (s) = ∫ f (− y) .e (− dy ) isy 2π ∞ −∞ 1 =− ∫ f (− y). e isy dy 2π ∞ [ ] ∞ 1 = ∫ f (− x). e dx = F f (− x) isx 2π −∞ 7. Transform of Derivatives If F(s) is the Fourier transform of f (x ) and if f (x) is continuous, f ′(x) is piecewise continuously differentiable, f (x ) and f ′(x) are absolutely integrable in (−∞ , ∞) and lim [ f ( x)] = 0 , then x → ±∞ F ( f ′( x ) ) = −is F ( s ) Proof: By the first three conditions given, F { f (x)} and F { f ′(x)} exist. ∞ 1 F { f ′( x)} = ∫ f ′( x) e isx dx 2π −∞ ∞ = 1 [e isx ] ∞ f ( x) −∞ − is ∫e isx f ( x) dx, on int egrating by parts. 2π 2π −∞ = 0 − isF { f ( x)} , by the given condition. = −is F ( s). The theorem can be extended as follows. If f , f ′, f ′′, , f ( n −1) are continuous, f (n ) is piecewise continuous, f , f ′, f ′′, , f ( n ) are absolutely integrable in (−∞ , ∞) and f , f ′, f ′′,  , f ( n −1) → 0 as x → ±∞ , then F ( f ( n ) ( x) ) = (−is ) n F ( s ) 13
  • 14. 8. Derivatives of the Transform dF ( s ) If F(s) is the Fourier transform of f (x ) then F [ x. f ( x )] = (−i ) ds Proof: ∞ 1 F (s) = ∫ f ( x )e isx dx 2π −∞ ∞ ∫ ds [ f ( x)e ]dx dF ( s ) 1 d ∴ = isx ds 2π −∞ ∞ i = ∫ [ x. f ( x)]e dx = iF [ xf ( x)] isx 2π −∞ dF ( s) ∴ ( −i ) = F [ x. f ( x)] ds [ ] Extending, we get, F x n . f ( x) = (−i ) n d n F (s) ds n DEFINITION ∞ 1 ∫ f ( x − u ) g (u )du is called the convolution product or simply the convolution 2π −∞ of the functions f (x) and g (x) and is denoted by f ( x) * g ( x ) . 9. Convolution Theorem If F(s) and G(s) are the Fourier transform of f (x ) and g (x) respectively then the Fourier transform of the convolution of f(x) and g(x) is the product of their Fourier transforms. i.e., F [ f ( x ) * g ( x)] = F ( s ).G ( s) Proof ∞ 1 F [ f ( x ) * g ( x )] = ∫∞ f ( x) * g ( x)e dx isx 2π − 1  1 ∞ ∞  isx = ∫∞  2π −∫∞ f ( x − u) g (u )du  e dx 2π −   1 ∞  1 ∞  = ∫∞  2π −∫∞ f ( x − u)e dxdu, isx g (u )  2π −  on changing the order of int egration. 14
  • 15. ∫ g (u )[e ] 1 = ius F ( s) du , by the shifting property. 2π −∞ ∞ 1 = F ( s). ∫ g (u ).e ius du 2π −∞ = F ( s).G ( s ) Inverting, we get F −1 [ F ( s ).G ( s)] = f ( x) * g ( x ) = F −1 { F ( s )} * F −1 { G ( s )} 10. Parseval’s Identity (or) Energy Theorem If f (x) is a given function defined in (−∞ , ∞) then it satisfy the identity, ∞ ∞ ∫ ∫ F ( s) 2 2 f ( x) dx = ds −∞ −∞ where F(s) is the Fourier transform of f (x ) . Proof: We know that F −1 [ F ( s ).G ( s)] = f ( x) * g ( x ) ∞ ∞ 1 1 ∫ F (s).G(s)e ∫ f (t ) g ( x − t )dt −isx ds = 2π −∞ 2π −∞ Putting x = 0, we get ∞ ∞ ∫ F (s).G(s)ds = ∫ f (t ) g (−t )dt −∞ −∞ ………………..(1) Let g ( − t ) = f (t ) .……………….(2) i.e., g (t ) = f (−t ) ………………..(3) ∴ G ( s ) = F [ f (− x)] = F ( s ) by property (9) i.e., ∴ G ( s) = F ( s) ………………..(4) Substituting (2) and (4) in (1) we get ∞ ∞ ∫ F (s).F (s) ds = −∞ ∫ f (t ). f (t ) dt −∞ [ F (s).F (s) = F (s) ] ∞ ∞ ∫ ∫ 2 2 2 F ( s) ds = f ( x) dx −∞ −∞ 11. If f (x) and g (x) are given functions of x and FC [ f ( x)] and FC [ g ( x)] are their Fourier cosine transforms and FS [ f ( x)] and FS [ g ( x)] are their Fourier sine transforms then 15
  • 16. ∞ ∞ (i) ∫ 0 f ( x ) g ( x)dx = ∫ FC [ f ( x)].FC [ g ( x)]ds = ∫ FS [ f ( x )].FS [ g ( x)]ds 0 0 ∞ ∞ ∞ ∫ f ( x) dx = ∫ FC [ f ( x)] ds = ∫ FS [ f ( x )] ds , 2 2 2 (ii) 0 0 0 which is Parseval’s identity for Fourier cosine and sine transforms. Proof: ∞ ∞  2∞  (i) ∫ FC [ f ( x )].FC [ g ( x )]ds = ∫ FC [ f ( x)]  ∫ g ( x) cos sx dx  ds 0 0  π 0  ∞  2∞  = ∫ g ( x)  ∫ FC [ f ( x)] cos sx ds  dx, 0  π 0  Changing the order of integration ∞ = ∫ f ( x) g ( x)dx 0 Similarly we can prove the other part of the result. (ii) Replacing g ( x) = f * ( x) in (i) and noting that FC [ f ( x)] = FC [ f ( x)] and FS [ f ( x)] = FS [ f ( x)] , we get ∞ ∞ ∞ ∫0 f ( x ). f ( x).dx = ∫ FC [ f ( x)].FC [ f ( x)] ds = ∫ FS [ f ( x )].FS [ f ( x )] ds 0 0 ∞ ∞ ∞ ∫ f ( x) .dx = ∫ FC [ f ( x)] ds = ∫ FS [ f ( x)] ds 2 2 2 i.e., 0 0 0 12. If FC [ f ( x)] = FC ( s ) and FS [ f ( x)] = FS ( s ) , then d (i) { FC ( s)} = − FS { xf ( x)} and ds d (ii) { FS ( s)} = − FC { xf ( x)}. ds Proof: ∞ 2 π∫ FC ( s) = f ( x) cos sx dx 0 ∞ d { FC ( s)} = ∫ f ( x)(− x sin sx)dx ds 0 ∞ = − ∫ {xf ( x)}sin sx dx 0 = − FS {xf ( x)} Similarly the result (ii) follows. 16
  • 17. PROBLEMS a 2 − x 2  x <a 1. Show that the Fourier transform of f ( x ) =  is 0  x >a>0 ∞ 2  sin as − as cos as  sin t − t cos t π 2   . Hence deduce that ∫ dt = . Using Parseval’s π s 3  0 t 3 4 ∞ 2  sin t − t cos t  π identity show that ∫  3  dt = . 0  t 15 Solution: We know that ∞ 1 F [ f ( x )] = ∫ f ( x).e isx dx 2π −∞ 1   −a a ∞ =  ∫ f ( x)e dx + ∫ f ( x)e dx + ∫ f ( x )e dx  isx isx isx 2π  −∞ −a a  1   a a 1 =  0 + ∫ ( a 2 − x 2 )e isx dx + 0 = ∫ (a 2 − x 2 ).e isx dx 2π  − a  2π −a a 1  2  e isx   e isx   e isx  =  ( a − x 2 )  is  − (−2 x) 2 2  − 2 3 3   i s  i s  2π        − a = 1  − 2a isa  2 e +e [−isa 2i ] [  + 3 e −isa − e isa  ] 2π  s s  1  − 2a 4  =  2 [2 cos as ] + 3 sin as  2π  s s  4  sin as − as cos as  = 2π   s3   2  sin as − as cos as  F (s) = 2 π  s3   2  sin s − s cos s  When a = 1, F ( s ) = 2 ………………..(A) π   s3   Using inverse Fourier Transform, we get 17
  • 18. 1 2 1 f ( x) = 2π .2. π ∫s −∞ 3 [sin as − as cos as].e −isx ds ∞ 1 2 1 { sin as − as cos as}{ cos sx − i sin sx} ds π −∫ s 3 = .2. 2π ∞ ∞ 2 sin as − as cos as π −∫ f ( x) = cos sx ds ∞ s3 [The second integral is odd and hence its value is zero] ∞ 4 sin as − as cos as = ∫ cos sx ds π 0 s3 [since the integrand is an even function of s] Putting a = 1, we get ∞ 4 sin s − s cos s f ( x) = ∫ cos sx ds π 0 s3 ∞ 4 sin t − t cos t f ( x) = ∫ cos tx dt π 0 t3 Putting x = 0, in the given function we get ∞ 4 sin t − t cos t π∫ dt = f (0) = 1 0 t3 ∞ sin t − t cos t π ∴ ∫ 0 t 3 dt = 4 ∞ ∞ ∫ ∫ F (s) 2 2 Using Parseval’s identity, f ( x) dx = ds [Using (A)] −∞ −∞ 2  2  ∞  2. (sin s − s cos s)  1 π ∫  − ∞ s3  ds = ∫ (1 − x 2 ) 2 dx  −1     ∞ 2 1 8  sin s − s cos s  ∫∞  ds = 2 ∫ (1 − x ) dx 2 2 π− s 3  0 ∞ 2 16  sin s − s cos s  8 ∫ π 0 s 3  ds = 2.  15 18
  • 19. 2  sin s − s cos s  π i.e., ∫ 0 s 3  ds =  15 ∞ 2  sin t − t cos t  π i.e., ∫ t3 0  dt =  15 2. Find the Fourier Transform of f (x) if 1 − x , x < 1  f ( x) =  0,  x >1 ∞ 4  sin t  π Hence deduce that ∫   dt = 0 t  3 Solution: We know that ∞ 1 F [ f ( x )] = ∫∞ f ( x).e dx isx 2π − 1 ∫ (1 − x ).e 1 = isx dx 2π −1 Since x > 1, f ( x) = 0, i.e., in − ∞ < x < −1, and 1 < x < ∞, f ( x ) = 0. 1 ∫ [1 − x ] [ cos sx + i sin sx] dx 1 = 2π −1 1 ∫ [1 − x ] cos sx dx 1 = 2π −1 The second integral becomes zero since it is an odd function. 1 2 = 2π ∫ (1 − x) cos sx dx 0 [ [1 − x ] cos x is an even function] 1 2   sin sx   − cos sx  = (1 − x ) s  − (−1) s 2  π      0 2  − cos s 1  =  + 2 π  s2 s  2 1 i.e., F ( s) = . (1 − cos s ) π s2 19
  • 20. Using Parseval’s identity ∞ ∞ ∫ ∫ F ( s) 2 2 f ( x) dx = ds −∞ −∞ 1 ∞ [1 − x ] dx = 2 ∫ (1 − cos s) ds 2 ∫ 2 −1 π −∞ s4 1 ∞ 2 (1 − cos s) 2 2 ∫ [1 − x ] dx = π −∫ 2 ds 0 ∞ s4 ∞ 2 2 (1 − cos s ) 2 3 π −∫ = ds ∞ s4 put s = 2t when s = ∞, t = ∞ ds = 2dt when s = −∞, t = −∞ π ∞ (1 − cos 2t ) 2 3 −∫ = .2 dt ∞ 16t 4 π ∞ (1 − cos 2t ) 2 3 −∫ = dt ∞ 8t 4 ∞ π (1 − cos 2t ) 2 = 2∫ dt 3 0 8t 4 π ∞ sin 4 t 3 ∫ t4 = dt 0 ∞ 4  sin t  π i.e., ∫  t  dt = 3 0  ∞ dx 3. Evaluate ∫ (x 0 2 + a )( x 2 + b 2 ) 2 using transforms. Solution: 2 a We know that the Fourier cosine transform of f ( x) = e − ax is . 2 . π s + a2 2 b Similarly the Fourier cosine transform of f ( x) = e − ax is . 2 . π s + b2 20
  • 21. ∞ We know that ∫ FC [ f ( x)].FC [ g ( x)] ds = ∫ f ( x).g ( x) dx 0 0 ∞ ∞ 2 a 2 b i.e., ∫ 0 . 2 . . 2 π s +a π s +b 2 2 . ds = ∫ e − ax .e −bx dx 0 ∞ ∞ 2 ab ∫ (s 2 + a 2 )(s 2 + b 2 ) ds = ∫ e −( a +b ) x i.e., ds π 0 0 ∞  e −( a + b ) x  1 1 =  = 0− =  − ( a + b)  0 − ( a + b) a + b ∞ dx π i.e., ∫ (x 0 2 2 2 2 = + a )( x + b ) 2ab( a + b) 4. Find the Fourier transform of e − a x and hence deduce that ∞ cos xt π −a x (i) ∫ 2 2 dt = e 0 a +t 2a [ (ii) F xe −a x ]=i 2 2as π (s + a 2 ) 2 2 Solution: ∞ 1 F [ f ( x )] = ∫ f ( x).e isx dx 2π −∞ 1   0 ∞ = ∫ f ( x)e isx dx + ∫ f ( x)e isx dx  2π  −∞ 0  e − ax  if 0 ≤ x < ∞ Here f ( x ) =  ax e  if − ∞ < x < 0 1  ax isx  0 ∞ = ∫ e. e dx + ∫ e −ax .e isx dx  2π  −∞ 0  1  ( a +is ) x  0 ∞ =  ∫ e. dx + ∫ e −( a −is ) x dx  2π  −∞ 0  21
  • 22. 1  e ( a +is ) x   e −( a −is ) x   0 ∞ =   +   2π  (a + is)  −∞  − (a − is )  0    1  1 1  =  a + is + a − is  2π   Fe[ −a x ]= 2 a π s + a2 2 Using inversion formula, we get ∞ 1 2 a f ( x) = 2π ∫ −∞ . 2 π s +a 2 e −isx ds ∞ a cos sx − i sin sx π −∫ = ds ∞ s2 + a2 ∞ a cos x = π−∫∞ s 2 + a 2 ds ∞ cos sx π π −a x ∫s 0 2 +a 2 dx = 2a f ( x) = 2a .e (or ) ∞ cos tx π −a x ∫s 0 2 +a 2 dt = 2a .e Putting a = 1, we get, Fe[ ]=−x 2 1 . π s2 +1 ∞ ∞ cos sx π −x cos tx π −x and ∫ s 2 + 1 ds = 2 e 0 (or ) ∫t 0 2 +1 dt = e 2 FINITE FOURIER TRANSFORMS If f (x) is a function defined in the interval (0 , l) then the finite Fourier sine transform of f (x) in 0 <x < l is defined as 22
  • 23. nπx l FS [ f ( x)] = ∫ f ( x). sin dx where ‘n’ is an integer 0 l The inverse finite Fourier sine transform of FS [ f (x )] is f (x) and is given by 2 ∞ nπx f ( x) = ∑ FS [ f ( x )] sin l n =1 l The finite Fourier cosine transform of f (x ) in 0 < x < l is defined as nπx l FC [ f ( x)] = ∫ f ( x). cos dx where ‘n’ is an integer 0 l The inverse finite Fourier cosine transform of FC [ f (x)] is f (x) and is given by 1 2 ∞ nπx f ( x) = FC (0) + ∑ FC [ f ( x )] cos l l n =1 l PROBLEMS 1. Find the finite Fourier sine and cosine transforms of f ( x) = x 2 in 0 < x < l. Solution: The finite Fourier sine transform is nπx l FS [ f ( x)] = ∫ f ( x). sin dx 0 l Here f ( x) = x 2 [ ] nπx l FS x 2 = ∫ x 2 . sin dx 0 l l   nπx   nπx   nπx    − cos   − sin   cos  = x 2 l  − 2 x l  + 2 l    nπ   n 2π 2   n 3π 3           l   l2   l 3  0  − l3 2l 3 2l 3 = cos nπ + 3 3 cos nπ − 3 3 nπ nπ nπ = l3 nπ 2l 3 [ (−1) n +1 + 3 3 (−1) n − 1 nπ ] The finite Fourier cosine transform is 23
  • 24. nπx l FC [ f ( x)] = ∫ f ( x). cos dx 0 l Here f ( x) = x 2 [ ] nπx l FC x 2 = ∫ x 2 . cos dx 0 l l   nπx   nπx   nπx    sin   − cos   − sin  = x 2  l  − 2 x l  + 2 l    nπ   n 2π 2   n 3π 3    l         l2   l3  0 2l 3 = cos nπ n 2π 2 2l 3 = (−1) n nπ 2 2 2. Find the finite Fourier sine and cosine transforms of f ( x ) = x in (0 , π ) . Solution: The finite Fourier sine transform of f ( x) = x in (0 , π ) is π FS [ f ( x)] = ∫ f ( x ). sin nx dx 0 Here f ( x) = x in (0 , π ) π π   − cos nx   − sin nx  FS [ x ] = ∫ x. sin nx dx =  x  − 1 2  0   n   n  0 π π =− cos nπ = (−1) n +1 . n n The finite Fourier cosine transform of f ( x) = x in (0 , π ) is π FC [ f ( x)] = ∫ f ( x). cos nx dx 0 Here f ( x) = x in (0 , π ) π π   sin nx   − cos nx  FC [ x ] = ∫ x. cos nx dx =  x  − 1 2  0   n   n  0 = 1 n 2 1 1 [ cos nπ − 2 = 2 (−1) n − 1 n n ] 24
  • 25. 2π (−1) p−1 3. Find f (x) if its finite sine transform is given by , where p is positive p3 integer and 0 < x < π . Solution: We know that the inverse Fourier sine transform is given by ∞ 2 f ( x) = ∑ FS [ f ( x )] sin px ………………..(1) π p =1 2π (−1) p−1 Here FS [ f (x )] = ………………..(2) p3 Substituting (2) in (1), we get 2 ∞ 2π (−1) p −1 f ( x) = ∑ sin px π p =1 p3 ∞ (−1) p −1 = 4∑ sin px p =1 p3  2 pπ  cos   3  find FC [ f ( p )] if 0 < x <1. −1 4. If f ( p) = (2 p + 1) 2 Solution: ∞ nπx We know that FC −1 [ f ( p)] = 1 FC (0) + 2 ∑ FC [ f ( x)] cos l l n =1 l  2 pπ  cos  Here f ( p) =  3  (2 p + 1) 2 Let FC [ f ( x)] = f ( p) ∞ nπx ∴ FC −1 [ f ( p)] = 1 f C ( 0) + 2 ∑ f ( p ) cos [ l = 1] l l n =1 l  2 pπ  ∞ cos  = 1 + 2∑  3  . cos nπx n =1 (2 p + 1) 2 25
  • 26. UNIT-4 PART A 1. State the Fourier integral theorem. Ans: If f (x) is a given function defined in (-l , l) and satisfies Dirichlet’s conditions, then ∞ ∞ 1 f ( x) = π ∫ ∫ f (t ) cos λ (t − x) dt dλ 0 −∞ 2. State the convolution theorem of the Fourier transform. Ans: If F(s) and G(s) are the Fourier transform of f (x ) and g (x) respectively then the Fourier transform of the convolution of f(x) and g(x) is the product of their Fourier transforms. i.e., F [ f ( x ) * g ( x)] = F ( s ).G ( s) 3. Write the Fourier transform pair. Ans: F [ f (x)] and F −1 [ F ( S )] are Fourier transform pairs. 4. Find the Fourier sine transform of f ( x) = e − ax (a > 0). Ans: ∞ 2 FS [ f ( x)] = π ∫ f ( x ). sin sx dx 0 2 ∞ 2 s   ∞ −ax b  ∫ e . sin sx dx =  ∫ e sin bx dx = 2 − ax =  π 0 π  s2 + a2     0 a + b2  2 s  = π  s2 + a2    5. If the Fourier transform of f (x ) is F(s) then prove that . F [ f ( x − a )] = e isa F ( s) Ans: ∞ 1 F [ f ( x − a)] = ∫∞ f ( x − a).e dx isx 2π − Put x-a = y dx = dy When x = −∞, y = −∞ and x = ∞, y = ∞ ∞ ∞ 1 e ias F [ f ( x − a)] = ∫ f ( y).e is ( y + a ) . dy = ∫ f ( y).e isy .dy 2π −∞ 2π −∞ ∞ e ias = ∫ f ( x).e .dx = e isa F ( s ) isx 2π −∞ 6. State the Fourier transforms of the derivatives of a function. 26